Формула параллельных сопротивлений: Онлайн-калькулятор расчета последовательного и параллельного соединения резисторов

Содержание

формула и примеры расчета сопротивления, напряжения, тока и мощности

При проектировании электрических схем возникает необходимость использования последовательного и параллельного соединений резисторов. Соединения применяются также и при ремонтах электрооборудования, поскольку в некоторых ситуациях невозможно найти эквивалентный номинал резистора. Выполнить расчет просто, и справиться с этой операцией может каждый.

Типы проводников

Проводимость веществом электрического тока связана с наличием в нем свободных носителей заряда. Их количество определяется по электронной конфигурации. Для этого необходима химическая формула вещества, при помощи которой можно вычислить их общее число. Значение для каждого элемента берется из периодической системы Дмитрия Ивановича Менделеева.

Электрический ток — упорядоченное движение свободных носителей заряда, на которые воздействует электромагнитное поле. При протекании тока по веществу происходит взаимодействие потока заряженных частиц с узлами кристаллической решетки, при этом часть кинетической энергии частицы превращается в тепловую энергию. Иными словами, частица «ударяется» об атом, а затем снова продолжает движение, набирая скорость под действием электромагнитного поля.

Процесс взаимодействия частиц с узлами кристаллической решетки называется электрической проводимостью или сопротивлением материала. Единицей измерения является Ом, а определить его можно при помощи омметра или расчитать. Согласно свойству проводимости, вещества можно разделить на 3 группы:

  1. Проводники (все металлы, ионизированный газ и электролитические растворы).
  2. Полупроводники (Si, Ge, GaAs, InP и InSb).
  3. Непроводники (диэлектрики или изоляторы).

Проводники всегда проводят электрический ток, поскольку содержат в своем атомарном строении свободные электроны, анионы, катионы и ионы. Полупроводники проводят электричество только при определенных условиях, которые влияют на наличие или отсутствие свободных электронов и дырок. К факторам, влияющим на проводимость, относятся следующие: температура, освещенность и т. д. Диэлектрики вообще не проводят электричество, поскольку в их структуре вообще отсутствуют свободные носители заряда. При выполнении расчетов каждый радиолюбитель должен знать зависимость сопротивления от некоторых физических величин.

Зависимость сопротивления

Значение электропроводимости зависит от нескольких факторов, которые необходимо учитывать при расчетах, изготовлении элементов резистивной нагрузки (резисторов), ремонте и проектировании устройств. К этим факторам необходимо отнести следующие:

  1. Температура окружающей среды и материала.
  2. Электрические величины.
  3. Геометрические свойства вещества.
  4. Тип материала, из которого изготовлен проводник (полупроводник).

К электрическим величинам можно отнести разность потенциалов (напряжение), электродвижущую силу (ЭДС) и силу тока. Геометрией проводника является его длина и площадь поперечного сечения.

Электрические величины

Зависимость величины электропроводимости от параметров электричества определяется законом Ома. Существует две формулировки: одна — для участка, а другая — для полной цепи. В первом случае соотношение определяются, исходя из значений силы тока (I) и напряжения (U) простой формулой: I = U / R. Из соотношения видна прямо пропорциональная зависимость тока от величины напряжения, а также обратно пропорциональная от сопротивления. Можно выразить R: R = U / I.

Для расчета электропроводимости всего участка следует воспользоваться соотношением между ЭДС (e), силой тока (i), а также внутренним сопротивлением источника питания (Rвн): i = e / (R+Rвн). В этом случае величина R вычисляется по формуле: R = (e / i) — Rвн. Однако при выполнении расчетов необходимо учитывать также геометрические параметры и тип проводника, поскольку они могут существенно повлиять на вычисления.

Тип и геометрические параметры

Свойство вещества к проводимости электричества определяется структурой кристаллической решетки, а также количеством свободных носителей. Исходя из этого, тип вещества является ключевым фактором, который определяет величину электропроводимости. В науке коэффициент, определяющий тип вещества, обозначается литерой «р» и называется удельным сопротивлением. Его значение для различных материалов (при температуре +20 градусов по Цельсию) можно найти в специальных таблицах.

Иногда для удобства расчетов используется обратная величина, которая называется удельной проводимостью (σ). Она связана с удельным сопротивлением следующим соотношением: p = 1 / σ. Площадь поперечного сечения (S) влияет на электрическое сопротивление. С физической точки зрения, зависимость можно понять следующим образом: при малом сечении происходят более частые взаимодействия частиц электрического тока с узлами кристаллической решетки.

Поперечное сечение можно вычислить по специальному алгоритму:

  1. Измерение геометрических параметров проводника (диаметр или длину сторон) при помощи штангенциркуля.
  2. Визуально определить форму материала.
  3. Вычислить площадь поперечного сечения по формуле, найденной в справочнике или интернете.

В случае когда проводник имеет сложную структуру, необходимо вычислить величину S одного элемента, а затем умножить результат на количество элементов, входящих в его состав. Например, если провод является многожильным, то следует вычислить S для одной жилы. После этого нужно умножить, полученную величину S, на количество жил. Зависимость R от вышеперечисленных величин можно записать в виде соотношения: R = p * L / S. Литера «L» является длиной проводника. Однако для получения точных расчетов необходимо учитывать температурные показатели внешней среды и проводника.

Температурные показатели

Существует доказательство зависимости удельного сопротивления материала от температуры, основанное на физическом эксперименте. Для проведения опыта нужно собрать электрическую цепь, состоящую из следующих элементов: источника питания, нихромовой спирали, соединительных проводов амперметра и вольтметра. Приборы нужны для измерения значений силы тока и напряжения соответственно. При протекании электричества происходит нагревание нихромовой пружины. По мере ее нагревания, показания амперметра уменьшаются. При этом происходит существенное падение напряжения на участке цепи, о котором свидетельствуют показания вольтметра.

В радиотехнике уменьшение величины напряжение называется просадкой или падением. Формула зависимости р от температуры имеет следующий вид: p = p0 * [1 + a * (t — 20)]. Значение p0 — удельное сопротивление материала, взятого из таблицы, а литера «t» — температура проводника.

Температурный коэффициент «а» принимает следующие значения: для металлов — a>0, а для электролитических растворов — a<0. Для получения формулы, определяющей все зависимости, необходимо подставить все соотношения в общую формулу зависимости R от типа материала, температуры, длины и сечения: R = p0 * [1 + a * (t — 20)] * L / S. Формулы используются только для расчетов и изготовления резисторов. Для быстрого измерения величины сопротивления применяется омметр.

Объединение резистивных радиокомпонентов

Для получения необходимого номинала сопротивления применяются два типа соединения резисторов: параллельное и последовательное. Если их соединить параллельно, то нужно два вывода одного резистора подключить к двум выводам другого. Если соединение является последовательным, то один вывод резистора соединяется с одним выводом другого резистора. Соединения используются для получения необходимых номиналов сопротивлений, а также для увеличения рассеивания мощности тока, протекающего по цепи.

Каждое из соединений обладает определенными характеристиками. Кроме того, последовательно или параллельно могут объединяться несколько резисторов. Соединения также могут быть смешанными, т. е. применяться оба типа объединения радиокомпонентов.

Параллельное соединение

При параллельном подключении значение напряжения на всех резисторах одинаковое, а сила тока — обратно пропорциональна их общему сопротивлению. В интернете web-разработчики создали для расчета величины общего сопротивления параллельного соединения резисторов онлайн-калькулятор.

Рассчитывается общее сопротивление при параллельном соединении по формуле: 1 / Rобщ = (1 / R1) + (1 / R2) + …+ (1 / Rn). Если выполнить математические преобразования и привести к общему знаменателю, то получится удобная формула параллельного соединения для расчета Rобщ. Она имеет следующий вид: Rобщ = (R1 * R2 * … * Rn) / (R1 + R2 + … + Rn). Если необходимо рассчитать величину Rобщ только для двух радиокомпонентов, то формула параллельного сопротивления имеет следующий вид: Rобщ = (R1 * R2) / (R1 + R2).

При ремонте или проектировании схемы устройства возникает задача объединения нескольких резистивных элементов для получения конкретной величины сопротивления. Например, значение Rобщ для определенной цепочки элементов равно 8 Ом, которое получено при расчетах. Перед радиолюбителем стоит задача, какие нужно подобрать номиналы для получения нужного значения (в стандартном ряду резисторов отсутствует радиокомпонент с номиналом в 8 Ом, а только 7,5 и 8,2). В этом случае нужно найти сопротивление при параллельном соединении резистивных элементов. Посчитать значение Rобщ для двух элементов можно следующим образом:

  1. Номинал резистора в 16 Ом подойдет.
  2. Подставить в формулу: R = (16 * 16) / (16 + 16) = 256 / 32 = 8 (Ом).

В некоторых случаях следует потратить больше времени на подбор необходимых номиналов. Можно применять не только два, но и три элемента. Сила тока вычисляется с использованием первого закона Кирхгофа. Формулировка закона следующая: общее значение тока, входящего и протекающего по цепи, равен выходному его значению. Величина силы тока для цепи, состоящей из двух резисторов (параллельное соединение) рассчитывается по такому алгоритму:

  1. Ток, протекающий через R1 и R2: I1 = U / R1 и I2 = U / R2 соответственно.
  2. Общий ток — сложение токов на резисторах: Iобщ = I1 + I2.

Например, если цепь состоит из 2 резисторов, соединенных параллельно, с номиналами в 16 и 7,5 Ом. Они запитаны от источника питания напряжением в 12 В. Значение силы тока на первом резисторе вычисляется следующим способом: I1 = 12 / 16 = 0,75 (А). На втором резисторе ток будет равен: I2 = 12 / 7,5 = 1,6 (А). Общий ток определяется по закону Кирхгофа: I = I1 + I2 = 1,6 + 0,75 = 2,35 (А).

Последовательное подключение

Последовательное включение резисторов также применяется в радиотехнике. Методы нахождения общего сопротивления, напряжения и тока отличаются от параллельного подключения. Основные правила соединения следующие:

  1. Ток не изменяется на участке цепи.
  2. Общее напряжение равно сумме падений напряжений на каждом резисторе.
  3. Rобщ = R1 + R2 + … + Rn.

Пример задачи следующий: цепочка, состоящая из 2 резисторов (16 и 7,5 Ом), питается от источника напряжением 12 В и током в 0,5 А. Необходимо рассчитать электрические параметры для каждого элемента. Порядок расчета следующий:

  1. I = I1 = I2 = 0,5 (А).
  2. Rобщ = R1 + R2 = 16 + 7,5 = 23,5 (Ом).
  3. Падения напряжения: U1 = I * R1 = 0,5 * 16 = 8 (В) и U2 = I * R2 = 0,5 * 7,5 = 3,75 (В).

Не всегда выполняется равенство напряжений (12 В не равно 8 + 3,75 = 11,75 В), поскольку при этом расчете не учитывается сопротивление соединительных проводов. Если схема является сложной, и в ней встречается два типа соединений, то нужно выполнять расчеты по участкам. В первую очередь, рассчитать для параллельного соединения, а затем для последовательного.

Таким образом, параллельное и последовательное соединения резисторов применяются для получения более точных значений сопротивлений, а также при отсутствии необходимого номинала радиокомпонента при проектировании или ремонте устройств.

Сопротивление при параллельном соединении, формула для расчета сопротивления при параллельном соединении

В этой статье мы разберем, как посчитать общее сопротивление при параллельном соединении сопротивлений. Параллельным соединением сопротивлений называется соединение (рисунок ниже), при котором один зажим каждого из сопротивлений присоединяется к одной точке (узлу) электрической цепи, а другой зажим каждого из тех же сопротивлений присоединяется к другой точке электрической цепи. Таким образом, между двумя точками (узлами) электрической цепи включается несколько сопротивлений, образующих параллельные ветви.

При этом напряжение между концами всех ветвей будет одним и тем же, а токи в отдельных ветвях определяются по закону Ома:
I1 = U / r1 ; I2 = U / r2 ; I3 = U / r3.

Напряжение U между узлами (А и Б):
U = I1r1 = I2r2 = I3r3,
откуда
I1 / I2 = R2 / R1  и  I2 / I3 = R3 / R2,
т. е.

Токи в параллельных ветвях распределяются обратно пропорционально их сопротивлениям.

Согласно первому правилу Кирхгофа,
I = I1 + I2 + I3
или
U / Rсум = U / R1 + U / R2 + U / R2 = U (1 / R1 + 1 / R2 + 1 / R3).
Произведя сокращение на U, получим:
1 / Rсум = 1 / R1 + 1 / R2 + 1 / R3
или
g = g1 + g2 + g3 ,

где R и g—сопротивление и проводимость разветвленной цепи или, как их часто называют, общие сопротивление и проводимость цепи.
Из полученной формулы следует, что

Общая проводимость разветвленной цепи равна сумме проводимостей отдельных ветвей.

Формула
1 / Rсум = 1 / R1 + 1 / R2 + 1 / R3
дает возможность определить общее сопротивление цепи. Например, для трех параллельно соединенных сопротивлений, приведя правую часть уравнения к общему знаменателю, получим:
1 / Rсум = R2R3 + R1R3 + R1R2 / R1R2R3
откуда
Rсум = R1R2R3 / R2R3 + R1R3 + R1R2
Если сопротивления R1 = R2 = R3, то общее сопротивление цепи:
Rсум = R1 / 3,
а в общем случае при n параллельных ветвях с равными сопротивлениями R1 :
Rсум = R1 / n
В случае двух параллельных ветвей:
1 / Rсум = 1 / R1 + 1 / R2
откуда
Rсум = R1R2 / R2R3 + R1R3

При параллельном соединении приемников энергии все они находятся под одним и тем же напряжением, и режим работы каждого из них не зависит от остальных. Совершенно иначе обстоит дело при последовательном соединении приемников, при котором изменение сопротивления одного из них тотчас же приводит к изменению напряжения на других, последовательно соединенных с ним. Поэтому электрические лампы и двигатели, предназначенные для работы при определенном (номинальном) напряжении, включаются параллельно. Одинаковые электрические лампы иногда соединяются последовательно. Пусть, например, напряжение сети U, а напряжение лампы U0 < U Тогда n ламп соединяются цепочкой друг за другом, причем n > U / U0
Такое соединение ламп можно встретить, например, в трамваях, метро и других случаях.

Пример 1:
К сети с напряжением 220 в параллельно подключены двигатель мощностью 1,1 квт и 11 ламп, каждая мощностью 40 вт. Определить ток в главных (подводящих) проводах

Ток двигателя
I1 = P1 / U = 1100 / 220 = 5a.
Ток ламп
I2 = P2 / U = 11 x 40 / 220 = 2a.
Ток в подводящих проводах
I = I1 + I2 = 5 + 2 = 7a.

Пример 2:
Определить общее сопротивление десяти параллельно включенных ламп накаливания, если каждая из них 240 ом:
R = Rл / n = 240 / 10 = 24ом.

Последовательное и параллельное соединение проводников — урок. Физика, 8 класс.

В быту и в промышленности в электрическую цепь соединяются сразу несколько потребителей электрической энергии. Различают три вида соединения сопротивлений (резисторов):

  1. последовательное соединение проводников
  2. параллельное соединение проводников
  3. смешанное соединение проводников

Последовательное соединение проводников

Схема соединения выглядит следующим образом:

 

 

Обрати внимание!

При последовательном соединении все входящие в него проводники соединяются друг за другом, т.е. конец первого проводника соединяется с началом второго.

 

 

Опыт показывает:

Сила тока в любых частях цепи одна и та же (об этом свидетельствуют показания амперметров): I=I1=I2.

Если выкрутить одну лампу, то цепь разомкнётся, а другая лампа тоже погаснет.

 

Опыт показывает следующее: 

При последовательном соединении сопротивлений результирующее напряжение равно сумме напряжений на участках: U=U1&plus;U2.

 

 

Результирующее сопротивление последовательно соединённых потребителей равно сумме сопротивлений потребителей: R=R1&plus;R2.


Для проверки данного утверждения можно использовать омметр. При подключении омметра ключ должен быть разомкнут!

 

Омметр подключают по очереди к каждому потребителю, а потом к обоим одновременно.

 

Сопротивление цепи \(R\), состоящей из \(n\) одинаковых ламп, сопротивлением R1 каждая, в \(n\) раз больше сопротивления одной лампы: \(R\) = R1* \(n\).

 

Параллельное соединение проводников

Схема соединения выглядит следующим образом:

 

 

Обрати внимание!

При параллельном соединении все входящие в него проводники одним своим концом присоединяются к одной точке цепи А, а вторым концом — к другой точке В.

 

 

Опыт доказывает:

Сила тока в неразветвлённой части цепи равна сумме сил тока в отдельных параллельно соединённых проводниках.

Об этом свидетельствуют показания амперметров: I=I1+I2.

 

 

Если выкрутить одну лампу, то другая лампа продолжает гореть. Это свойство используют для подключения бытовых приборов в помещении.

 

Опыт свидетельствует, что:

Напряжение на участке цепи АВ и на концах всех параллельно соединённых проводников одно и то же.

Об этом свидетельствуют показания вольтметров:

U=U1=U2.

 

 

Общее сопротивление цепи при параллельном соединении проводников определяется по формуле:

 

1R=1R1&plus;1R2.

 

Обратное значение общего сопротивления равно сумме обратных значений сопротивлений отдельных проводников.

Для проверки формулы можно использовать омметр. При подключении омметра ключ должен быть разомкнут!

 

Сопротивление цепи \(R\), состоящей из \(n\) одинаковых ламп, сопротивлением R1 каждая, в \(n\) раз меньше сопротивления одной лампы: \(R\) = R1/ \(n\).

Источники:

http://files.school-collection.edu.ru/dlrstore/669ba073-e921-11dc-95ff-0800200c9a66/3_17.swf
http://files.school-collection.edu.ru/dlrstore/669ba074-e921-11dc-95ff-0800200c9a66/3_18.swf

http://class-fizika.narod.ru/8_33.htm

Соединение резисторов — Основы электроники

Соединение резисторов в различные конфигурации очень часто применяются в электротехнике и электронике.
Здесь мы будем рассматривать только участок цепи, включающий в себя соединение резисторов.
Соединение резисторов может производиться последовательно, параллельно и смешанно (то есть и последовательно и параллельно), что показано на рисунке 1.

Рисунок 1. Соединение резисторов.

Последовательное соединение резисторов

Последовательное соединение резисторов это такое соединение, в котором конец одного резистора соединен с началом второго резистора, конец второго резистора с началом третьего и так далее (рисунок 2).

Рисунок 2. Последовательное соединение резисторов.

То есть при последовательном соединении резисторы подключатся друг за другом. При таком соединении через резисторы будет протекать один общий ток.
Следовательно, для последовательного соединения резисторов будет справедливо сказать, что между точками А и Б есть только один единственный путь протекания тока.
Таким образом, чем больше число последовательно соединенных резисторов, тем большее сопротивление они оказывают протеканию тока, то есть общее сопротивление Rобщ возрастает.
Рассчитывается общее сопротивление последовательно соединенных резисторов по следующей формуле:

Rобщ = R1 + R2 + R3+…+ Rn.

Параллельное соединение резисторов

Параллельное соединение резисторов это соединение, в котором начала всех резисторов соединены в одну общую точку (А), а концы в другую общую точку (Б) (см. рисунок 3).

Рисунок 3. Параллельное соединение резисторов.

При этом по каждому резистору течет свой ток. При параллельном соединении при протекании тока из точки А в точку Б, он имеет несколько путей.
Таким образом, увеличение числа параллельно соединенных резисторов ведет к увеличению путей протекания тока, то есть к уменьшению противодействия протеканию тока. А это значит, чем большее количество резисторов соединить параллельно, тем меньше станет значение общего сопротивления такого участка цепи (сопротивления между точкой А и Б.)
Общее сопротивление параллельно соединенных резисторов определяется следующим отношением:

1/Rобщ= 1/R1+1/R2+1/R3+…+1/Rn

Следует отметить, что здесь действует правило «меньше — меньшего». Это означает, что общее сопротивление всегда будет меньше сопротивления любого параллельно включенного резистора.
Общее сопротивление для двух параллельно соединенных резисторов рассчитывается по следующей формуле:

Rобщ= R1*R2/R1+R2

Если имеет место два параллельно соединенных резистора с одинаковыми сопротивлениями, то их общее сопротивление будет равно половине сопротивления одного из них.

Смешанное соединение резисторов

Смешанное соединение резисторов является комбинацией последовательного и параллельного соединения. Иногда подобную комбинацию называют последовательно-параллельным соединением.
На рисунке 4 показан простейший пример смешанного соединения резисторов.

Рисунок 4. Смешанное соединение резисторов.

На этом рисунке видно, что резисторы R2 R3 соединены параллельно, а R1, комбинация R2 R3 и R4 последовательно.
Для расчета сопротивления таких соединений, всю цепь разбивают на простейшие участки, из параллельно или последовательно соединенных резисторов. Далее следуют следующему алгоритму:
1. Определяют эквивалентное сопротивление участков с параллельным соединением резисторов.
2. Если эти участки содержат последовательно соединенные резисторы, то сначала вычисляют их сопротивление.
3. После расчета эквивалентных сопротивлений резисторов перерисовывают схему. Обычно получается цепь из последовательно соединенных эквивалентных сопротивлений.
4. Рассчитывают сопротивления полученной схемы.

Пример расчета участка цепи со смешанным соединением резисторов приведен на рисунке 5.

Рисунок 5. Расчет сопротивления участка цепи при смешанном соединении резисторов.

ПОНРАВИЛАСЬ СТАТЬЯ? ПОДЕЛИСЬ С ДРУЗЬЯМИ В СОЦИАЛЬНЫХ СЕТЯХ!

Похожие материалы:

Добавить комментарий

Как найти параллельное сопротивление формула

Из закона Ома и первого и второго правил Кирхгофа следует:

При параллельном соединении величина обратная полному сопротивлению, равна сумме величин, обратных сопротивлений ветвей.

При параллельном соединении полное сопротивление цепи меньше самого малого из сопротивлений ветвей.

Поскольку 1/R = G, т.е. проводимость, то
при параллельном соединении электрические проводимости отдельных ветвей складываются

Параллельное соединение двух сопротивлений

При параллельном соединении двух сопротивлений формула (1) упрощается

Параллельное соединение двух сопротивлений

При параллельном соединении двух сопротивлений формула (1) упрощается

Сопротивление проводников. Параллельное и последовательное соединение проводников.

Электри́ческое сопротивле́ние — физическая величина, характеризующая свойства проводника препятствовать прохождению электрического тока и равная отношениюнапряжения на концах проводника к силе тока, протекающего по нему [1] . Сопротивление для цепей переменного тока и для переменных электромагнитных полей описывается понятиями импеданса и волнового сопротивления. Сопротивлением (резистором) также называют радиодеталь, предназначенную для введения в электрические цепи активного сопротивления.

Сопротивление (часто обозначается буквой R или r) считается, в определённых пределах, постоянной величиной для данного проводника; её можно рассчитать как

U — разность электрических потенциалов (напряжение) на концах проводника;

I — сила тока, протекающего между концами проводника под действием разности потенциалов.

При последовательном соединении проводников (рис. 1.9.1) сила тока во всех проводниках одинакова:

Последовательное соединение проводников

По закону Ома, напряжения U1 и U2 на проводниках равны

Общее напряжение U на обоих проводниках равно сумме напряжений U1 и U2:

где R – электрическое сопротивление всей цепи. Отсюда следует:

При последовательном соединении полное сопротивление цепи равно сумме сопротивлений отдельных проводников.

Этот результат справедлив для любого числа последовательно соединенных проводников.

При параллельном соединении (рис. 1.9.2) напряжения U1 и U2 на обоих проводниках одинаковы:

Сумма токов I1 + I2, протекающих по обоим проводникам, равна току в неразветвленной цепи:

Этот результат следует из того, что в точках разветвления токов (узлы A и B) в цепи постоянного тока не могут накапливаться заряды. Например, к узлу A за время Δt подтекает заряд IΔt, а утекает от узла за то же время заряд I1Δt + I2Δt. Следовательно,I = I1 + I2.

Параллельное соединение проводников

Записывая на основании закона Ома

где R – электрическое сопротивление всей цепи, получим

При параллельном соединении проводников величина, обратная общему сопротивлению цепи, равна сумме величин, обратных сопротивлениям параллельно включенных проводников.

Этот результат справедлив для любого числа параллельно включенных проводников.

Формулы для последовательного и параллельного соединения проводников позволяют во многих случаях рассчитывать сопротивление сложной цепи, состоящей из многих резисторов. На рис. 1.9.3 приведен пример такой сложной цепи и указана последовательность вычислений.

Расчет сопротивления сложной цепи. Сопротивления всех проводников указаны вомах (Ом)

Следует отметить, что далеко не все сложные цепи, состоящие из проводников с различными сопротивлениями, могут быть рассчитаны с помощью формул для последовательного и параллельного соединения. На рис. 1.9.4 приведен пример электрической цепи, которую нельзя рассчитать указанным выше методом.

Пример электрической цепи, которая не сводится к комбинации последовательно и параллельно соединенных проводников

Параллельное соединение резисторов — одно из двух видов электрических соединений, когда оба вывода одного резистора соединены с соответствующими выводами другого резистора или резисторов. Зачастую резисторы соединяют последовательно или параллельно для того, чтобы создать более сложные электронные схемы.

Схема параллельного соединения резисторов показан на рисунке ниже. При параллельном соединении резисторов, напряжение на всех резисторах будет одинаковым, а протекающий через них ток будет пропорционален их сопротивлению:

Формула параллельного соединения резисторов

Общее сопротивление нескольких резисторов соединенных параллельно определяется по следующей формуле:

Ток, протекающий через отдельно взятый резистор, согласно закону Ома, можно найти по формуле:

Параллельное соединение резисторов — расчет

Пример №1

При разработке устройства, возникла необходимость установить резистор с сопротивлением 8 Ом. Если мы просмотрим весь номинальный ряд стандартных значений резисторов, то мы увидим, что резистора с сопротивлением в 8 Ом в нем нет.

Выходом из данной ситуации будет использование двух параллельно соединенных резисторов. Эквивалентное значение сопротивления для двух резисторов соединенных параллельно рассчитывается следующим образом:

Данное уравнение показывает, что если R1 равен R2, то сопротивление R составляет половину сопротивления одного из двух резисторов. При R = 8 Ом, R1 и R2 должны, следовательно, иметь значение 2 × 8 = 16 Ом.
Теперь проведем проверку, рассчитав общее сопротивление двух резисторов:

Таким образом, мы получили необходимое сопротивление 8 Ом, соединив параллельно два резистора по 16 Ом.

Пример расчета №2

Найти общее сопротивление R из трех параллельно соединенных резисторов:

Общее сопротивление R рассчитывается по формуле:

Этот метод расчета может быть использованы для расчета любого количества отдельных сопротивлений соединенных параллельно.

Один важный момент, который необходимо запомнить при расчете параллельно соединенных резисторов – это то, что общее сопротивление всегда будет меньше, чем значение наименьшего сопротивления в этой комбинации.

Как рассчитать сложные схемы соединения резисторов

Более сложные соединения резисторов могут быть рассчитаны путем систематической группировки резисторов. На рисунке ниже необходимо посчитать общее сопротивление цепи, состоящей из трех резисторов:


Для простоты расчета, сначала сгруппируем резисторы по параллельному и последовательному типу соединения.
Резисторы R2 и R3 соединены последовательно (группа 2). Они в свою очередь соединены параллельно с резистором R1 (группа 1).

Последовательное соединение резисторов группы 2 вычисляется как сумма сопротивлений R2 и R3:

В результате мы упрощаем схему в виде двух параллельных резисторов. Теперь общее сопротивление всей схемы можно посчитать следующим образом:

Расчет более сложных соединений резисторов можно выполнить используя законы Кирхгофа.

Ток, протекающий в цепи параллельно соединенных резисторах

Общий ток I протекающий в цепи параллельных резисторов равняется сумме отдельных токов, протекающих во всех параллельных ветвях, причем ток в отдельно взятой ветви не обязательно должен быть равен току в соседних ветвях.

Несмотря на параллельное соединение, к каждому резистору приложено одно и то же напряжение. А поскольку величина сопротивлений в параллельной цепи может быть разной, то и величина протекающего тока через каждый резистор тоже будет отличаться (по определению закона Ома).

Рассмотрим это на примере двух параллельно соединенных резисторов. Ток, который течет через каждый из резисторов ( I1 и I2 ) будет отличаться друг от друга поскольку сопротивления резисторов R1 и R2 не равны.
Однако мы знаем, что ток, который поступает в цепь в точке «А» должен выйти из цепи в точке «B» .

Первое правило Кирхгофа гласит: «Общий ток, выходящий из цепи равен току входящий в цепь».

Таким образом, протекающий общий ток в цепи можно определить как:

Затем с помощью закона Ома можно вычислить ток, который протекает через каждый резистор:

Ток, протекающий в R1 = U ÷ R1 = 12 ÷ 22 кОм = 0,545 мА

Ток, протекающий в R 2 = U ÷ R2 = 12 ÷ 47 кОм = 0,255 мА

Таким образом, общий ток будет равен:

I = 0,545 мА + 0,255 мА = 0,8 мА

Это также можно проверить, используя закон Ома:

I = U ÷ R = 12 В ÷ 15 кОм = 0,8 мА (то же самое)

где 15кОм — это общее сопротивление двух параллельно соединенных резисторов (22 кОм и 47 кОм)

И в завершении хочется отметить, что большинство современных резисторов маркируются цветными полосками и назначение ее можно узнать здесь.

Параллельное соединение резисторов — онлайн калькулятор

Чтобы быстро вычислить общее сопротивление двух и более резисторов, соединенных параллельно, вы можете воспользоваться следующим онлайн калькулятором:

Подведем итог

Когда два или более резистора соединены так, что оба вывода одного резистора соединены с соответствующими выводами другого резистора или резисторов, то говорят, что они соединены между собой параллельно. Напряжение на каждом резисторе внутри параллельной комбинации одинаковое, но токи, протекающие через них, могут отличаться друг от друга, в зависимости от величины сопротивлений каждого резистора.

Эквивалентное или полное сопротивление параллельной комбинации всегда будет меньше минимального сопротивления резистора входящего в параллельное соединение.

Параллельное соединение резисторов | Электротехника

Параллельное соединение резисторов. При параллельном соединении резисторов нескольких приемников они включаются между двумя точками электрической цепи, образуя параллельные ветви (рис. 26, а). Заменяя

Рис. 26. Схемы параллельного соединения приемников

лампы резисторами с сопротивлениями R1, R2, R3, получим схему, показанную на рис. 26, б.
При параллельном соединении ко всем резисторам приложено одинаковое напряжение U. Поэтому согласно закону Ома:

I1=U/R1; I2=U/R2; I3=U/R3.

Ток в неразветвленной части цепи согласно первому закону Кирхгофа I = I1+I2+I3, или

I = U / R1 + U / R2 + U / R3 = U (1/R1 + 1/R2 + 1/R3) = U / Rэк (23)

Следовательно, эквивалентное сопротивление рассматриваемой цепи при параллельном соединении трех резисторов определяется формулой

1/Rэк = 1/R1 + 1/R2 + 1/R3 (24)

Вводя в формулу (24) вместо значений 1/Rэк, 1/R1, 1/R2 и 1/R3 соответствующие проводимости Gэк, G1, G2 и G3, получим: эквивалентная проводимость параллельной цепи равна сумме проводимостей параллельно соединенных резисторов:

Gэк = G1+ G2 +G3 (25)

Таким образом, при увеличении числа параллельно включаемых резисторов результирующая проводимость электрической цепи увеличивается, а результирующее сопротивление уменьшается.
Из приведенных формул следует, что токи распределяются между параллельными ветвями обратно пропорционально их электрическим сопротивлениям или прямо пропорционально их проводимостям. Например, при трех ветвях

I1 : I2 : I3 = 1/R1 : 1/R2 : 1/R3 = G1 + G2 + G3 (26)

В этом отношении имеет место полная аналогия между распределением токов по отдельным ветвям и распределением потоков воды по трубам.
Приведенные формулы дают возможность определить эквивалентное сопротивление цепи для различных конкретных случаев. Например, при двух параллельно включенных резисторах результирующее сопротивление цепи

Rэк=R1R2/(R1+R2)

при трех параллельно включенных резисторах

Rэк=R1R2R3/(R1R2+R2R3+R1R3)

При параллельном соединении нескольких, например n, резисторов с одинаковым сопротивлением R1 результирующее сопротивление цепи Rэк будет в n раз меньше сопротивления R1, т.е.

Rэк = R1 / n (27)

Проходящий по каждой ветви ток I1, в этом случае будет в п раз меньше общего тока:

I1 = I / n (28)

При параллельном соединении приемников, все они находятся под одним и тем же напряжением, и режим работы каждого из них не зависит от остальных. Это означает, что ток, проходящий по какому-либо из приемников, не будет оказывать существенного влияния на другие приемники. При всяком выключении или выходе из строя любого приемника остальные приемники остаются включенными. Поэтому параллельное соединение имеет существенные преимущества перед последовательным, вследствие чего оно получило наиболее широкое распространение. В частности, электрические лампы и двигатели, предназначенные для работы при определенном (номинальном) напряжении, всегда включают параллельно.
На электровозах постоянного тока и некоторых тепловозах тяговые двигатели в процессе регулирования скорости движения нужно включать под различные напряжения, поэтому они в процессе разгона переключаются с последовательного соединения на параллельное.

Соединение резисторов. Типы соединений и формулы расчёта общего сопротивления резисторов.

Как правильно соединять резисторы?

О том, как соединять конденсаторы и рассчитывать их общую ёмкость уже рассказывалось на страницах сайта. А как соединять резисторы и посчитать их общее сопротивление? Именно об этом и будет рассказано в этой статье.

Резисторы есть в любой электронной схеме, причём их номинальное сопротивление может отличаться не в 2 – 3 раза, а в десятки и сотни раз. Так в схеме можно найти резистор на 1 Ом, и тут же неподалёку на 1000 Ом (1 кОм)!

Поэтому при сборке схемы либо ремонте электронного прибора может потребоваться резистор с определённым номинальным сопротивлением, а под рукой такого нет. В результате быстро найти подходящий резистор с нужным номиналом не всегда удаётся. Это обстоятельство тормозит процесс сборки схемы или ремонта. Выходом из такой ситуации может быть применение составного резистора.

Для того чтобы собрать составной резистор нужно соединить несколько резисторов параллельно или последовательно и тем самым получить нужное нам номинальное сопротивление. На практике это пригождается постоянно. Знания о правильном соединении резисторов и расчёте их общего сопротивления выручают и ремонтников, восстанавливающих неисправную электронику, и радиолюбителей, занятых сборкой своего электронного устройства.

Последовательное соединение резисторов.

В жизни последовательное соединение резисторов имеет вид:


Последовательно соединённые резисторы серии МЛТ

Принципиальная схема последовательного соединения выглядит так:

На схеме видно, что мы заменяем один резистор на несколько, общее сопротивление которых равно тому, который нам необходим.

Подсчитать общее сопротивление при последовательном соединении очень просто. Нужно сложить все номинальные сопротивления резисторов входящих в эту цепь. Взгляните на формулу.

Общее номинальное сопротивление составного резистора обозначено как Rобщ.

Номинальные сопротивления резисторов включённых в цепь обозначаются как R1, R2, R3,…RN.

Применяя последовательное соединение, стоит помнить одно простое правило:

Из всех резисторов, соединённых последовательно главную роль играет тот, у которого самое большое сопротивление. Именно он в значительной степени влияет на общее сопротивление.

Что это значит?

Так, например, если мы соединяем три резистора, номинал которых равен 1, 10 и 100 Ом, то в результате мы получим составной на 111 Ом. Если убрать резистор на 100 Ом, то общее сопротивление цепочки резко уменьшиться до 11 Ом! А если убрать, к примеру, резистор на 10 Ом, то сопротивление будет уже 101 Ом. Как видим, резисторы с малыми сопротивлениями в последовательной цепи практически не влияют на общее сопротивление.

Параллельное соединение резисторов.

Можно соединять резисторы и параллельно:


Два резистора МЛТ-2, соединённых параллельно

Принципиальная схема параллельного соединения выглядит следующим образом:

Для того чтобы подсчитать общее сопротивление нескольких параллельно соединённых резисторов понадобиться знание формулы. Выглядит она вот так:

Эту формулу можно существенно упростить, если применять только два резистора. В таком случае формула примет вид:

Есть несколько простых правил, позволяющих без предварительного расчёта узнать, каково должно быть сопротивление двух резисторов, чтобы при их параллельном соединении получить то, которое требуется.

Если параллельно соединены два резистора с одинаковым сопротивлением, то общее сопротивление этих резисторов будет ровно в два раза меньше, чем сопротивление каждого из резисторов, входящих в эту цепочку.

Это правило исходит из простой формулы для расчёта общего сопротивления параллельной цепи, состоящей из резисторов одного номинала. Она очень проста. Нужно разделить номинальное сопротивление одного из резисторов на общее их количество:

Здесь R1 – номинальное сопротивление резистора. N – количество резисторов с одинаковым номинальным сопротивлением.

Ознакомившись с приведёнными формулами, вы скажите, что все они справедливы для расчёта ёмкости параллельно и последовательно соединённых конденсаторов. Да, только в отношении конденсаторов всё действует с точностью до «наоборот”. Узнать подробнее о соединении конденсаторов можно здесь.

Проверим справедливость показанных здесь формул на простом эксперименте.

Возьмём два резистора МЛТ-2 на 3 и 47 Ом и соединим их последовательно. Затем измерим общее сопротивление получившейся цепи цифровым мультиметром. Как видим оно равно сумме сопротивлений резисторов, входящих в эту цепочку.


Замер общего сопротивления при последовательном соединении

Теперь соединим наши резисторы параллельно и замерим их общее сопротивление.


Измерение сопротивления при параллельном соединении

Как видим, результирующее сопротивление (2,9 Ом) меньше самого меньшего (3 Ом), входящего в цепочку. Отсюда вытекает ещё одно известное правило, которое можно применять на практике:

При параллельном соединении резисторов общее сопротивление цепи будет меньше наименьшего сопротивления, входящего в эту цепь.

Что ещё нужно учитывать при соединении резисторов?

Во-первых, обязательно учитывается их номинальная мощность. Например, нам нужно подобрать замену резистору на 100 Ом и мощностью 1 Вт. Возьмём два резистора по 50 Ом каждый и соединим их последовательно. На какую мощность рассеяния должны быть рассчитаны эти два резистора?

Поскольку через последовательно соединённые резисторы течёт один и тот же постоянный ток (допустим 0,1 А), а сопротивление каждого из них равно 50 Ом, тогда мощность рассеивания каждого из них должна быть не менее 0,5 Вт. В результате на каждом из них выделится по 0,5 Вт мощности. В сумме это и будет тот самый 1 Вт.

Данный пример достаточно грубоват. Поэтому, если есть сомнения, стоит брать резисторы с запасом по мощности.

Подробнее о мощности рассеивания резистора читайте тут.

Во-вторых, при соединении стоит использовать однотипные резисторы, например, серии МЛТ. Конечно, нет ничего плохого в том, чтобы брать разные. Это лишь рекомендация.

Главная &raquo Радиоэлектроника для начинающих &raquo Текущая страница

Также Вам будет интересно узнать:

 

Как рассчитать последовательные и параллельные резисторы — Kitronik Ltd

Резисторы серии

Когда резисторы подключаются друг за другом, это называется последовательным соединением. Это показано ниже. Чтобы рассчитать общее общее сопротивление ряда резисторов, подключенных таким образом, вы складываете отдельные сопротивления. Это делается по следующей формуле: Rtotal = R1 + R2 + R3 и так далее. Пример: чтобы рассчитать полное сопротивление для этих трех последовательно соединенных резисторов.
Rtotal = R1 + R2 + R3 = 100 + 82 + 1 Ом = 183 Ом

Задача 1:

Рассчитайте общее сопротивление следующего последовательно включенного резистора.
R Итого = _______________
= _______________
R Итого = _______________
= _______________
R Итого = _______________
= _______________

Параллельные резисторы

Когда резисторы подключаются друг к другу (бок о бок), это называется параллельным подключением.Это показано ниже.

Два параллельных резистора

Чтобы рассчитать общее полное сопротивление двух резисторов, подключенных таким образом, вы можете использовать следующую формулу:
Пример: чтобы рассчитать полное сопротивление для этих двух резисторов, включенных параллельно.

Задача 2:

Рассчитайте полное сопротивление следующего резистора, включенного параллельно.

Три или более резистора параллельно

Для расчета общего общего сопротивления ряда из трех или более резисторов, подключенных таким образом, вы можете использовать следующую формулу: Пример: Чтобы вычислить общее сопротивление для этих трех резисторов, подключенных параллельно

Задача 3:

Рассчитайте полное сопротивление следующего резистора, включенного параллельно.

Ответы

Задача 1

1 = 1492 Ом 2 = 2242 Ом 3 = 4847 Ом

Задача 2

1 = 5 Ом 2 = 9,57 Ом 3 = 248,12 Ом

Задача 3

1 = 5,95 Ом 2 = 23,76 Ом Загрузите pdf-версию этой страницы здесь. Подробнее об авторе подробнее »

© Kitronik Ltd — Вы можете распечатать эту страницу и ссылку на нее, но не должны копировать страницу или ее часть без предварительного письменного согласия Kitronik.

Калькулятор параллельных резисторов

R1 + R2 = эквивалентный резистор R схема сопротивления, эквивалентная общая сумма резисторов, упрощенная комбинация = параллельная

параллельная калькуляция резисторов R1 + R2 = эквивалентный резистор R эквивалентная схема сопротивления, полная поисковая схема резистора, упрощенная совмещенная = параллельная — sengpielaudio Sengpiel Berlin


R всего Формула:
R всего = R1 × R2 / (R1 + R2)

Введите два значения резистора , будет рассчитано третье значение параллельной цепи.
Вы даже можете ввести общее сопротивление R общее и одно известное сопротивление R 1 или R 2 .

Формула (уравнение) для расчета двух сопротивлений R 1 и R 2 , соединенных параллельно:

Расчет необходимого параллельного резистора R 2 , при R 1 и общее сопротивление R дается всего :

Решение формулы R всего = ( R 1 × R 2 ) / ( R 1 + R 2 ) для R 1 1 Первый шаг — очистить все дроби путем умножения на наименьшее значение
. общий знаменатель, то есть R t × R 1 × R 2 … итого получаем:
1/ R итого = 1/ R 1 + 1/ R 2
R итого × R 1 × R 2 [1/ R всего = 1/ R 1 + 1/ R 2 ]
R 1 × R 2 = всего × R 2 + R всего × R 1 затем соберите члены с помощью R 1 и решите
R 1 × R 2 9 — R всего × R 1 = R всего × R 2
R 1 ( R 2 R всего ) = R 2 × R итого 9011 9
Последний шаг:
R 1 = R 2 × R всего / ( R всего )
или:
R 2 = R 1 ×
R 1 R всего )

Примечание: Этот калькулятор также может решать другие математические задачи.Расчет резисторов параллельно
точно так же, как и вычисления, необходимые для параллельных катушек индуктивности или последовательно включенных конденсаторов.

Два резистора, включенных параллельно, и результирующее общее сопротивление: Два одинаковых значения,
также покажите уравнение, что результаты всегда равны половине. Это упрощает работу, когда
проектирование схем или прототипирование. С кепками всегда вдвое больше, потом с кепками всего
просто сложите параллельно.

• Сопротивления поиска R 1 и R 2 , когда заданное сопротивление (эквивалентное сопротивление) известно •

Расчет: пары резисторов — вычислитель с обратной конструкцией
Поиск R 1 и R 2 с известным целевым сопротивлением

● Рассчитать несколько резисторов параллельно ●

Этот калькулятор определяет сопротивление от до 10 параллельно включенных резисторов .
Введите сопротивления в поля ниже и, когда все значения будут введены,
нажмите кнопку «рассчитать», и результат появится в поле под этой кнопкой.
В качестве теста, если мы введем сопротивления 4, 6 и 12 Ом, ответ должен быть 2 Ом.
Примечание. При снятии флажков вручную сохраненные значения не сбрасываются. Воспользуйтесь «сбросом».

Закон Ома — калькулятор и формулы

Два резистора, включенных параллельно, и результирующее общее сопротивление
Сопротивление в диапазоне от 1 Ом до 100 Ом

R2 R1
1 1.5 2,2 3,3 4,7 6,8 10 15 22 33 47 68
1 0,5 0,6 0,69 0.77 0,83 0,87 0,91 0,93 0,95 0,97 0,98 0,99
1,5 0,6 0,75 0,89 1.03 1,14 1,22 1,30 1,36 1,40 1,43 1.45 1,46
2,2 0,69 0,89 1,1 1,32 1,50 1,66 1,82 1,92 2,0 2,06 2,10 2,13
3,3 0,77 1.03 1,32 1.65 1,94 2,22 2,48 2,70 2,87 3,00 3,08 3,14
4,7 0,83 1,14 1,50 1,94 2,35 2,78 3,20 3,58 3,87 4,12 4.27 4,39
6,8 0,87 1,22 1,66 2,22 2,78 3,40 4,05 4,68 5,19 5,64 5,94 6,18
10 0,91 1,30 1,82 2.48 3,20 4,05 5,0 6,0 6,9 7,7 8,3 8,7
15 0,93 1,36 1,92 2,70 3,58 4,68 6,0 7,50 8,9 10,3 11,4 12.2
22 0,95 1,40 2,00 2,87 3,87 5,19 6,9 8,9 11,0 13,2 15,0 16,6
33 0,97 1,43 2,06 3,0 4.12 5,64 7,7 10,3 13,2 16,5 19,4 22,2
47 0,98 1,45 2,1 3,08 4,27 5,94 8,3 11,4 15,0 19,4 23,5 27.8
68 0,99 1,46 2,13 3,14 4,39 6,18 8,7 12,2 16,6 22,2 27,8 34,0

Примечание: Этот калькулятор также может решать другие математические задачи. Расчет резисторов параллельно
точно так же, как и вычисления, необходимые для параллельных катушек индуктивности или последовательно включенных конденсаторов.

Мощность, рассеиваемая в резисторе: P = В × I , P = В 2 / R , P = I 2 × 901.

Примечание: Для последовательно соединенных резисторов ток одинаков для каждого резистора,
а для резисторов, включенных параллельно, напряжение одинаково для каждого резистора.

Формулы и калькулятор »Электроника

Формулы, расчеты и калькулятор для определения общего сопротивления резисторов, установленных последовательно и параллельно.


Resistance Tutorial:
Что такое сопротивление Закон Ома Омические и неомические проводники Сопротивление лампы накаливания Удельное сопротивление Таблица удельного сопротивления для распространенных материалов Температурный коэффициент сопротивления Электрическая проводимость Последовательные и параллельные резисторы Таблица параллельных резисторов


Резисторы могут быть размещены во многих конфигурациях в электрической или электронной схеме — иногда последовательно, иногда параллельно.

Когда они размещаются в этих конфигурациях, важно иметь возможность рассчитать общее сопротивление. Этого можно довольно легко достичь, если использовать правильные формулы — есть простые формулы как для последовательных, так и для параллельных резисторов.

При проектировании электронной схемы или по другой причине возможность вычисления сопротивления комбинации резисторов может быть очень полезной.

В электронных схемах комбинации резисторов могут быть сведены к последовательным элементам и параллельным элементам, хотя при использовании других электронных компонентов комбинации могут быть более сложными.Однако во многих случаях расчет значений последовательного и параллельного сопротивления имеет большое значение.

Резисторы последовательно

Самая простая конфигурация электронной схемы — это резисторы, включенные последовательно. Это может произойти, если несколько этих электронных компонентов соединены последовательно, или необходимо добавить сопротивление кабеля к сопротивлению резистора и т. Д.

Если резисторы соединены последовательно, то общее сопротивление является просто суммой отдельных резисторов.

Последовательные резисторы

Величину резисторов или сопротивлений, включенных последовательно, можно математически выразить следующим образом:

Пример расчета последовательных резисторов:
В качестве примера, если три резистора, имеющие номиналы 1 кОм, 2 кОм и 3 кОм, соединены последовательно, то общее сопротивление составит 1 + 2 + 3 кОм = 6 кОм.

В реальных жизненных ситуациях и аспектах проектирования электрических и электронных схем будет много областей, где есть электронные компоненты, такие как резисторы или другие элементы, вносящие сопротивление, где количество последовательно соединенных сопротивлений, которые необходимо суммировать.

Сопротивления параллельно

Также есть много случаев, когда электронные компоненты, такие как резисторы, а также другие элементы, вызывающие сопротивление, появляются в электрической или электронной цепи параллельно.

Если резисторы размещены параллельно, они разделяют ток, и ситуацию немного сложнее вычислить, но все же довольно легко.

1Rtotal = 1R1 + 1R2 + 1R3 + ……

Пример расчета сопротивления резисторов, включенных параллельно:
Чтобы дать пример, если есть три резистора, подключенных параллельно со значениями 1 кОм, 2 кОм и Омега и 3 кОм, то можно вычислить общее значение комбинации:

1 / R Итого = 1/1000 + 1/2000 + 1/3000

1 / R Итого = 1/1000 + 1/2000 + 1/3000

1 / R Итого = 6/6000 + 3/6000 + 2/6000

1 / R Итого = 11/6000

R Всего = 6000/11 Ом или 545 Ом

Корпус только двух резисторов, включенных параллельно

Во многих конструкциях электронных схем наиболее распространенный пример параллельного подключения резисторов состоит только из двух электронных компонентов.

Часто бывает так, что один резистор подключается параллельно другому. Или другой случай может быть, когда резистор помещается на клеммы для цепи или сети, которая имеет определенное сопротивление. В этом случае необходимо только рассчитать общее сопротивление для двух параллельно включенных резисторов.

Если необходимо рассчитать общее значение для двух параллельных резисторов, уравнением можно манипулировать и значительно упростить его, как показано ниже:

Эта формула значительно упрощает вычисление номинала двух параллельно включенных резисторов, поскольку требует только одного умножения, одного сложения и одного деления.Часто это можно сделать мысленно или на клочке бумаги. В качестве альтернативы можно использовать наш простой калькулятор для двух параллельно включенных резисторов, приведенный ниже.

Калькулятор для двух резисторов, включенных параллельно

Этот калькулятор параллельного сопротивления обеспечивает простой метод расчета общего сопротивления для двух резисторов, соединенных параллельно.

Хотя вычисление номиналов параллельных резисторов для двух резисторов упрощается до простой формулы, иногда гораздо проще и быстрее использовать калькулятор.

Чтобы использовать калькулятор параллельных резисторов, просто введите значения параллельных резисторов в Ом, Ом или кОм и т. Д. В два поля ввода, но обратите внимание, что все значения должны быть в одних и тех же единицах, то есть оба в Ом кОм МОм и т. Д. Затем вычислитель параллельных резисторов предоставит общее сопротивление двух резисторов в тех же единицах, что и вход.

Введите два значения для резисторов, R1 и R2, в поля, представленные в калькуляторе ниже, нажмите вычислить, и будет предоставлено общее сопротивление.


Калькулятор параллельного сопротивления

Калькулятор параллельных резисторов позволяет легко рассчитать сопротивление двух параллельных резисторов, не записывая все и прибегая к ручке и бумаге или калькулятору в той или иной форме.

Знание того, как рассчитать значения резисторов, включенных последовательно и параллельно, является ключом к пониманию того, как работают электрические и электронные схемы. Эти концепции используются как вторая натура при проектировании электрических и электронных схем.

Другие основные концепции электроники:
Напряжение Текущий Мощность Сопротивление Емкость Индуктивность Трансформеры Децибел, дБ Законы Кирхгофа Q, добротность РЧ шум
Вернуться в меню «Основные понятия электроники». . .

Серия

и параллельные резисторы

  • Изучив этот раздел, вы сможете:
  • Рассчитайте значения общего сопротивления в цепях с последовательным сопротивлением.
  • Используйте соответствующие формулы для расчета сопротивления в цепях с параллельным сопротивлением.
  • • Вычисление суммы обратных величин.
  • • Произведение над суммой.
  • Рассчитайте значения общего сопротивления в последовательных / параллельных сетях.

Расчеты в последовательно- и параллельных резисторных цепях

Компоненты, включая резисторы в цепи, могут быть соединены вместе двумя способами:

ПОСЛЕДОВАТЕЛЬНО, так что один и тот же ток течет через все компоненты, но на каждом из них может существовать разная разность потенциалов (напряжение).

ПАРАЛЛЕЛЬНО, так что одна и та же разность потенциалов (напряжение) существует на всех компонентах, но каждый компонент может проводить разный ток.

Рис. 4.2.1 Резисторы серии

Рис. 4.2.2 Параллельные резисторы

В любом случае (для резисторов) полное сопротивление той части цепи, которая содержит резисторы, может быть рассчитано с использованием методов, описанных ниже.

Возможность рассчитать суммарное (общее) значение резисторов таким способом позволяет легко вычислить неизвестные значения сопротивления, тока и напряжения для довольно сложных схем, используя относительно простые методы.Это очень полезно при поиске неисправностей.

ПЕРЕД ДАЛЬНЕЙШЕЙ ДАЛЬНЕЙШЕЙ ПОПРАКТИКОЙ ИСПОЛЬЗУЙТЕ ФОРМУЛЫ ДЛЯ РАСЧЕТА ОБЩИХ ЗНАЧЕНИЙ СЕРИИ И ПАРАЛЛЕЛЬНЫХ РЕЗИСТОРОВ.

Для резисторов в серии:

Общее сопротивление двух или более резисторов, подключенных последовательно , определяется простым сложением индивидуальных значений резисторов, чтобы найти общую сумму (R TOT ):

Для резисторов, включенных параллельно:

Для расчета общего сопротивления цепи, в которой используются параллельные резисторы, можно использовать следующую формулу.

Обратите внимание, однако, что эта формула НЕ дает вам общего сопротивления R TOT . Это дает вам ВЗАИМОДЕЙСТВИЕ R TOT или:

Это совсем другое значение — и НЕ является полным сопротивлением. Он равен 1, разделенному на рэндов ИТОГО . Чтобы получить правильное значение для R TOT (которое будет обратным 1 / R TOT , т. Е. TOT /1, просто нажмите соответствующую клавишу на вашем калькуляторе (отмеченную 1 / x или x-1) .

Другой способ расчета параллельных цепей.

Суммарное сопротивление двух резисторов, включенных параллельно , которое не включает обратные, определяется по формуле:

Эту формулу часто называют «произведение над суммой».

Он рассчитывает только ДВА резистора параллельно? Ну да, но это не большая проблема. Если имеется более двух параллельных резисторов, просто выберите два из них и определите общее сопротивление для этих двух — затем используйте это общее сопротивление, как если бы это был один резистор, и составьте еще одну пару с третьим резистором.Определите новую сумму и так далее, пока вы не включите все параллельные резисторы в этой конкретной сети.

О, еще кое-что, что нужно помнить о произведении над суммой, видите скобки вокруг суммы (нижняя часть) формулы? Это означает, что вы должны решить это, прежде чем использовать его для разделения продукта (верхняя часть) на. Если вы этого не сделаете, ваш ответ будет неправильным.

Звучит сложно? Не совсем, это просто вопрос повторения, и на практике вы не часто встречаетесь с множеством параллельных сетей с гораздо более чем двумя резисторами.Тем не менее, какую формулу вы выберете, зависит от вас, взаимная или сумма продукта.

Подсказки

Использование обратного метода

Если вы используете ВЗАИМНЫЙ МЕТОД для параллельных цепей, НЕ ЗАБУДЬТЕ, когда вы добавили обратные величины отдельных резисторов — вы должны снова найти обратную величину. 1 / R1 + 1 / R2 + 1 / R3 = 1 / R TOT , и чтобы найти R TOT , вы должны найти обратную величину 1 / R TOT .

Упрощающие схемы

Для комбинированных последовательных и параллельных цепей сначала разработайте секцию цепи (последовательную или параллельную).Затем перерисуйте схему, заменив участок, сопротивление которого вы нашли, одним резистором. Теперь у вас есть упрощенная схема, по которой можно найти R TOT .

Вы можете использовать формулу «произведение на сумму»:

Для цепей с более чем двумя параллельными резисторами просто выработайте два параллельных резистора одновременно, используя формулу произведения на сумму, а затем перерисуйте схему, заменив два резистора одним резистором, значение которого является объединенным сопротивлением двух .

Теперь вы можете использовать первое комбинированное значение в качестве единственного резистора со следующим параллельным резистором и так далее. Таким образом, можно выработать большое количество параллельных резисторов с использованием произведения на сумму.

Когда все параллельные резисторы одинакового номинала.

Если несколько одинаковых параллельных резисторов подключены, общее сопротивление будет равно номиналу резистора, умноженному на обратную величину количества резисторов.

, т.е. два параллельных резистора 12K имеют общее сопротивление

12K x 1/2 = 6K

Три параллельно включенных резистора 12 кОм имеют суммарное сопротивление

12K x 1/3 = 4K и т. Д.

Проверяю ответ

Суммарное значение любого количества параллельных резисторов всегда будет МЕНЬШЕ, чем значение наименьшего отдельного резистора в сети. Используйте этот факт, чтобы проверить свои ответы.

Серия

и параллельная комбинация

Попробуйте несколько вычислений, основанных на последовательной и параллельной цепях резисторов. Для этого вам просто нужно использовать информацию на этой странице и на странице «Советы по расчету резисторов». Вас просят вычислить общее сопротивление для каждой цепи.Вы можете выбрать, какую формулу использовать

Вы также можете получить помощь по математике, загрузив нашу бесплатную брошюру «Советы по математике».

Прежде чем начать, подумайте об этих нескольких советах. Они упростят задачу, если вы будете внимательно им следовать.

1. Разработайте ответы с помощью карандаша и бумаги; перерисуйте схему, над которой работаете.

2. Конечно, ответ — это не просто число, это будет определенное количество Ом, не забудьте указать правильную единицу (например.грамм. Ω, KΩ или MΩ) или ваш ответ не имеет смысла.

3. Когда вы вводите значения в калькулятор, преобразуйте все значения КОм или МОм в Ом с помощью клавиши EXP. Если вы здесь ошибетесь, то получите действительно глупые ответы, в тысячи раз слишком большие или слишком маленькие.

Итак, вы прочитали эти инструкции и готовы приступить к работе. Вот способ решить типичную проблему на бумаге, чтобы (со временем) вы не запутались.

Пример последовательной и параллельной цепей

.

Хорошо, есть что вспомнить, так почему бы не попробовать несколько практических вопросов в модуле резисторов 4.5 по определению общего сопротивления некоторых цепей резисторов?

резисторов последовательно и параллельно

Резисторы серии

Общее сопротивление в цепи с последовательно включенными резисторами равно сумме отдельных сопротивлений.

Цели обучения

Рассчитайте общее сопротивление в цепи с последовательно включенными резисторами

Основные выводы

Ключевые моменты
  • Один и тот же ток последовательно проходит через каждый резистор.
  • Отдельные последовательно включенные резисторы не получают полное напряжение источника, а делят его.
  • Общее сопротивление в последовательной цепи равно сумме отдельных сопротивлений: [латекс] \ text {RN} (\ text {series}) = \ text {R} _1 + \ text {R} _2 + \ text {R} _3 +… + \ text {R} _ \ text {N} [/ latex].
Ключевые термины
  • серия : ряд элементов, которые следуют одно за другим или связаны друг за другом.
  • сопротивление : Противодействие прохождению электрического тока через этот элемент.

Обзор

Большинство схем имеет более одного компонента, называемого резистором, который ограничивает поток заряда в цепи. Мера этого предела для потока заряда называется сопротивлением. Самыми простыми комбинациями резисторов являются последовательное и параллельное соединение. Общее сопротивление комбинации резисторов зависит как от их индивидуальных значений, так и от способа их подключения.

Последовательные цепи : Краткое введение в анализ последовательных и последовательных цепей, включая закон Кирхгофа по току (KCL) и закон Кирхгофа по напряжению (KVL).

Резисторы серии

Резисторы

включены последовательно всякий раз, когда поток заряда или ток должен проходить через компоненты последовательно.

Резисторы в серии : Эти четыре резистора подключены последовательно, потому что, если бы ток подавался на один конец, он бы протекал через каждый резистор последовательно до конца.

показывает резисторы, последовательно подключенные к источнику напряжения. Общее сопротивление в цепи равно сумме отдельных сопротивлений, поскольку ток должен проходить через каждый резистор последовательно через цепь.

Резисторы, подключенные последовательно : три резистора, подключенные последовательно к батарее (слева), и эквивалентное одиночное или последовательное сопротивление (справа).

Использование закона Ома для расчета изменений напряжения в резисторах серии

В соответствии с законом Ома падение напряжения V на резисторе при протекании через него тока рассчитывается по формуле V = IR, где I — ток в амперах (A), а R — сопротивление в омах (Ω). .

Таким образом, падение напряжения на R 1 равно V 1 = IR 1 , на R 2 равно V 2 = IR 2 , а на R 3 равно V 3 = IR 3 .Сумма напряжений будет равна: V = V 1 + V 2 + V 3 , исходя из сохранения энергии и заряда. Если подставить значения отдельных напряжений, получим:

[латекс] \ text {V} = \ text {IR} _1 + \ text {IR} _2 + \ text {IR} _3 [/ latex]

или

[латекс] \ text {V} = \ text {I} (\ text {R} _1 + \ text {R} _2 + \ text {R} _3) [/ латекс]

Это означает, что полное сопротивление в серии равно сумме отдельных сопротивлений. Следовательно, для каждой цепи с Н количество резисторов, включенных последовательно:

[латекс] \ text {RN} (\ text {series}) = \ text {R} _1 + \ text {R} _2 + \ text {R} _3 +… + \ text {R} _ \ text {N }.[/ латекс]

Поскольку весь ток должен проходить через каждый резистор, он испытывает сопротивление каждого, и последовательно соединенные сопротивления просто складываются.

Поскольку напряжение и сопротивление имеют обратную зависимость, отдельные последовательно включенные резисторы не получают полное напряжение источника, а делят его. Об этом свидетельствует пример, когда две лампочки соединены в последовательную цепь с аккумулятором. В простой схеме, состоящей из одной батареи 1,5 В и одной лампочки, падение напряжения на лампе будет равно 1.5V через него. Однако, если бы две лампочки были соединены последовательно с одной и той же батареей, на каждой из них было бы падение напряжения 1,5 В / 2 или 0,75 В. Это будет очевидно по яркости света: каждая из двух последовательно соединенных лампочек будет в два раза слабее, чем одиночная лампочка. Следовательно, резисторы, соединенные последовательно, потребляют такое же количество энергии, как и один резистор, но эта энергия распределяется между резисторами в зависимости от их сопротивлений.

Параллельные резисторы

Общее сопротивление в параллельной цепи равно сумме обратных сопротивлений каждого отдельного сопротивления.

Цели обучения

Рассчитайте общее сопротивление в цепи с параллельно включенными резисторами

Основные выводы

Ключевые моменты
  • Общее сопротивление в параллельной цепи меньше наименьшего из отдельных сопротивлений.
  • Каждый резистор, включенный параллельно, имеет то же напряжение, что и приложенный к нему источник (напряжение в параллельной цепи постоянно).
  • Параллельные резисторы не получают суммарный ток каждый; они делят его (ток зависит от номинала каждого резистора и общего количества резисторов в цепи).
Ключевые термины
  • сопротивление : Противодействие прохождению электрического тока через этот элемент.
  • параллельно : расположение электрических компонентов, при котором ток течет по двум или более путям.

Обзор

Резисторы в цепи могут быть включены последовательно или параллельно. Общее сопротивление комбинации резисторов зависит как от их индивидуальных значений, так и от способа их подключения.

Parallel Circuits : Краткий обзор анализа параллельных цепей с использованием таблиц VIRP для школьников-физиков.

Параллельные резисторы

Резисторы

включены параллельно, когда каждый резистор подключен непосредственно к источнику напряжения путем соединения проводов с незначительным сопротивлением. Таким образом, к каждому резистору приложено полное напряжение источника.

Параллельное соединение резисторов : Параллельное соединение резисторов.

Каждый резистор потребляет такой же ток, как если бы он был единственным резистором, подключенным к источнику напряжения. Это верно для схем в доме или квартире. Каждая розетка, подключенная к устройству («резистор»), может работать независимо, и ток не должен проходить через каждое устройство последовательно.

Закон и параллельные резисторы

На каждый резистор в цепи подается полное напряжение. Согласно закону Ома токи, протекающие через отдельные резисторы, равны [латекс] \ text {I} _1 = \ frac {\ text {V}} {\ text {R} _1} [/ latex], [latex] \ text {I} _2 = \ frac {\ text {V}} {\ text {R} _2} [/ latex] и [latex] \ text {I} _3 = \ frac {\ text {V}} {\ text {R} _3} [/ латекс].Сохранение заряда подразумевает, что полный ток складывается из этих токов: 90 · 109

Параллельные резисторы : Три резистора, подключенных параллельно батарее, и эквивалентное одиночное или параллельное сопротивление.

[латекс] \ text {I} = \ text {I} _1 + \ text {I} _2 + \ text {I} _3. [/ Latex]

Подстановка выражений для отдельных токов дает:

[латекс] \ text {I} = \ frac {\ text {V}} {\ text {R} _1} + \ frac {\ text {V}} {\ text {R} _2} + \ frac {\ текст {V}} {\ text {R} _3} [/ latex]

или

[латекс] \ text {I} = \ text {V} (\ frac {1} {\ text {R} _1} + \ frac {1} {\ text {R} _2} + \ frac {1} { \ text {R} _3}) [/ latex]

Это означает, что полное сопротивление в параллельной цепи равно сумме обратных сопротивлений каждого отдельного сопротивления.Следовательно, для каждой цепи с числом [latex] \ text {n} [/ latex] или параллельно подключенных резисторов

[латекс] \ text {R} _ {\ text {n} \; (\ text {parallel})} = \ frac {1} {\ text {R} _1} + \ frac {1} {\ text { R} _2} + \ frac {1} {\ text {R} _3}… + \ frac {1} {\ text {R} _ \ text {n}}. [/ Latex]

Это соотношение приводит к общему сопротивлению, которое меньше наименьшего из отдельных сопротивлений. Когда резисторы подключены параллельно, от источника течет больше тока, чем протекает для любого из них по отдельности, поэтому общее сопротивление ниже.

Каждый резистор, включенный параллельно, имеет такое же полное напряжение источника, как на него, но делит общий ток между ними. Примером может служить соединение двух лампочек в параллельную цепь с аккумулятором на 1,5 В. В последовательной цепи две лампочки будут вдвое менее тусклыми при подключении к одному источнику батареи. Однако, если бы две лампочки были подключены параллельно, они были бы столь же яркими, как если бы они были подключены к батарее по отдельности. Поскольку к обеим лампочкам подается одинаковое полное напряжение, батарея также разряжается быстрее, поскольку она по существу обеспечивает полную энергию обеими лампочками.В последовательной цепи батарея будет работать столько же, сколько и с одной лампочкой, только тогда яркость будет разделена между лампочками.

Комбинированные схемы

Комбинированная цепь может быть разбита на аналогичные части, которые работают последовательно или параллельно.

Цели обучения

Описать расположение резисторов в комбинированной цепи и его практическое значение

Основные выводы

Ключевые моменты
  • Более сложные соединения резисторов иногда представляют собой просто комбинации последовательного и параллельного.
  • Различные части комбинированной схемы могут быть идентифицированы как последовательные или параллельные, уменьшены до их эквивалентов, а затем уменьшены до тех пор, пока не останется единственное сопротивление.
  • Сопротивление в проводах снижает ток и мощность, подаваемые на резистор. Если сопротивление в проводах относительно велико, как в изношенном (или очень длинном) удлинительном шнуре, то эти потери могут быть значительными и влиять на выходную мощность в устройствах.
Ключевые термины
  • серия : ряд элементов, которые следуют одно за другим или связаны друг за другом.
  • параллельно : расположение электрических компонентов, при котором ток течет по двум или более путям.
  • Комбинированная схема : электрическая цепь, содержащая несколько резисторов, соединенных в комбинации как последовательного, так и параллельного соединения.

Комбинированные схемы

Более сложные соединения резисторов иногда представляют собой просто комбинации последовательного и параллельного. Это часто встречается, особенно если учитывать сопротивление проводов.В этом случае сопротивление провода включено последовательно с другими сопротивлениями, включенными параллельно.

Комбинированная цепь может быть разбита на аналогичные части, которые являются последовательными или параллельными, как показано на схеме. На рисунке общее сопротивление может быть вычислено путем соединения трех резисторов друг с другом последовательно или параллельно. R 1 и R 2 соединены параллельно по отношению друг к другу, поэтому мы знаем, что для этого подмножества сопротивление, обратное сопротивлению, будет равно:

Сеть резисторов : В этой комбинированной схеме цепь может быть разбита на последовательный компонент и параллельный компонент.

Комбинированные схемы : Два параллельных резистора, соединенные последовательно с одним резистором.

[латекс] \ frac {1} {\ text {R} _1} + \ frac {1} {\ text {R} _2} [/ latex] или [латекс] \ frac {\ text {R} _1 \ text {R} _2} {\ text {R} _1 + \ text {R} _2} [/ latex]

R 3 соединен последовательно с как R 1 , так и R 2 , поэтому сопротивление будет рассчитываться как:

[латекс] \ text {R} = \ frac {\ text {R} _1 \ text {R} _2} {\ text {R} _1 + \ text {R} _2} + \ text {R} _3 [/ latex ]

Сложные комбинированные схемы

Для более сложных комбинированных схем различные части могут быть идентифицированы как последовательные или параллельные, уменьшены до их эквивалентов, а затем уменьшены до тех пор, пока не останется единственное сопротивление, как показано на.На этом рисунке комбинация из семи резисторов была идентифицирована как включенные последовательно или параллельно. На исходном изображении две обведенные кружком секции показывают резисторы, включенные параллельно.

Сокращение комбинированной схемы : Эта комбинация из семи резисторов имеет как последовательные, так и параллельные части. Каждое из них идентифицируется и приводится к эквивалентному сопротивлению, а затем уменьшается до тех пор, пока не будет достигнуто одно эквивалентное сопротивление.

Уменьшение этих параллельных резисторов до одного значения R позволяет нам визуализировать схему в более упрощенном виде.На верхнем правом изображении мы видим, что обведенная кружком часть содержит два последовательно соединенных резистора. Мы можем дополнительно уменьшить это до другого значения R, добавив их. Следующий шаг показывает, что два обведенных резистора включены параллельно. Уменьшение тех бликов, что последние два соединены последовательно и, таким образом, могут быть уменьшены до одного значения сопротивления для всей цепи.

Одним из практических следствий комбинированной схемы является то, что сопротивление в проводах снижает ток и мощность, подаваемую на резистор.Комбинированная цепь может быть преобразована в последовательную цепь на основе понимания эквивалентного сопротивления параллельных ветвей комбинированной цепи. Последовательная цепь может использоваться для определения общего сопротивления цепи. По сути, сопротивление провода является последовательным с резистором. Таким образом, увеличивается общее сопротивление и уменьшается ток. Если сопротивление провода относительно велико, как в изношенном (или очень длинном) удлинителе, то эти потери могут быть значительными. Если потребляется большой ток, падение ИК-излучения в проводах также может быть значительным.

Зарядка аккумулятора: последовательные и параллельные ЭДС

При последовательном включении источников напряжения их ЭДС и внутренние сопротивления складываются; параллельно они остаются прежними.

Цели обучения

Сравните сопротивления и электродвижущие силы для источников напряжения, подключенных с одинаковой и противоположной полярностью, последовательно и параллельно

Основные выводы

Ключевые моменты
  • ЭДС, соединенные последовательно с одинаковой полярностью, являются аддитивными и приводят к более высокой полной ЭДС.
  • Две ЭДС, соединенные последовательно с противоположной полярностью, имеют общую ЭДС, равную разнице между ними, и могут использоваться для зарядки источника более низкого напряжения.
  • Два источника напряжения с идентичными ЭДС, соединенные параллельно, имеют чистую ЭДС, эквивалентную одному источнику ЭДС, однако общее внутреннее сопротивление меньше и, следовательно, дает более высокий ток.
Ключевые термины
  • параллельно : расположение электрических компонентов, при котором ток течет по двум или более путям.
  • электродвижущая сила : (ЭДС) — напряжение, генерируемое батареей или магнитной силой в соответствии с законом Фарадея. Она измеряется в вольтах, а не в ньютонах, и поэтому на самом деле не является силой.
  • серия : ряд элементов, которые следуют одно за другим или связаны друг за другом.

Когда используется более одного источника напряжения, они могут быть подключены последовательно или параллельно, аналогично резисторам в цепи.Когда источники напряжения включены последовательно в одном направлении, их внутренние сопротивления складываются, а их электродвижущая сила или ЭДС складываются алгебраически. Эти типы источников напряжения распространены в фонариках, игрушках и других приборах. Обычно ячейки включены последовательно, чтобы обеспечить большую суммарную ЭДС.

Фонарик и лампочка : Последовательное соединение двух источников напряжения в одном направлении. Эта схема представляет собой фонарик с двумя последовательно включенными ячейками (источниками напряжения) и одной лампочкой (сопротивление нагрузки).

Батарея — это соединение нескольких гальванических элементов. Однако недостатком такого последовательного соединения ячеек является то, что их внутреннее сопротивление увеличивается. Иногда это может быть проблематично. Например, если вы поместите в машину две батареи на 6 В вместо обычной батареи на 12 В, вы добавите как ЭДС, так и внутреннее сопротивление каждой батареи. Таким образом, у вас будет такая же ЭДС 12 В, хотя внутреннее сопротивление тогда будет удвоено, что вызовет у вас проблемы, когда вы захотите запустить двигатель.

Но если ячейки противостоят друг другу — например, когда одна вставляется в прибор задом наперед, — общая ЭДС меньше, так как это алгебраическая сумма отдельных ЭДС. Когда он перевернут, он создает ЭДС, которая противодействует другой, и приводит к разнице между двумя источниками напряжения.

Зарядное устройство : представляет два источника напряжения, соединенных последовательно с противоположными ЭДС. Ток течет в направлении большей ЭДС и ограничивается суммой внутренних сопротивлений.(Обратите внимание, что каждая ЭДС представлена ​​на рисунке буквой E.) Зарядное устройство, подключенное к аккумулятору, является примером такого подключения. Зарядное устройство должно иметь большую ЭДС, чем батарея, чтобы через него протекал обратный ток.

Когда два источника напряжения с идентичными ЭДС соединены параллельно и также подключены к сопротивлению нагрузки, общая ЭДС равна индивидуальной ЭДС. Но общее внутреннее сопротивление уменьшается, поскольку внутренние сопротивления параллельны. Таким образом, параллельное соединение может производить больший ток.

Две идентичные ЭДС : Два источника напряжения с одинаковыми ЭДС (каждый помечен буквой E), соединенные параллельно, создают одинаковую ЭДС, но имеют меньшее общее внутреннее сопротивление, чем отдельные источники. Параллельные комбинации часто используются для подачи большего тока.

ЭДС и напряжение на клеммах

Выходное напряжение или напряжение на клеммах источника напряжения, такого как аккумулятор, зависит от его электродвижущей силы и внутреннего сопротивления.

Цели обучения

Выразите взаимосвязь между электродвижущей силой и напряжением на клеммах в форме уравнения

Основные выводы

Ключевые моменты
  • Электродвижущая сила (ЭДС) — это разность потенциалов источника при отсутствии тока.
  • Напряжение на клеммах — это выходное напряжение устройства, измеренное на его клеммах.
  • Напряжение на клеммах рассчитывается по формуле V = ЭДС — Ir.
Ключевые термины
  • электродвижущая сила : (ЭДС) — напряжение, генерируемое батареей или магнитной силой в соответствии с законом Фарадея. Она измеряется в вольтах, а не в ньютонах, и поэтому на самом деле не является силой.
  • напряжение на клеммах : выходное напряжение устройства, измеренное на его клеммах.
  • разность потенциалов : разница в потенциальной энергии между двумя точками в электрическом поле; разница в заряде между двумя точками в электрической цепи; Напряжение.

Когда вы забываете выключить автомобильные фары, они постепенно тускнеют по мере разрядки аккумулятора. Почему они просто не мигают, когда батарея разряжена? Их постепенное затемнение означает, что выходное напряжение батареи уменьшается по мере разряда батареи. Причина снижения выходного напряжения для разряженных или перегруженных батарей заключается в том, что все источники напряжения состоят из двух основных частей — источника электрической энергии и внутреннего сопротивления.

Электродвижущая сила

Все источники напряжения создают разность потенциалов и могут подавать ток, если подключены к сопротивлению. В небольшом масштабе разность потенциалов создает электрическое поле, которое воздействует на заряды, вызывая ток. Мы называем эту разность потенциалов электродвижущей силой (сокращенно ЭДС). ЭДС — это вообще не сила; это особый тип разности потенциалов источника при отсутствии тока. Единицы измерения ЭДС — вольты.

Электродвижущая сила напрямую связана с источником разности потенциалов, например с конкретной комбинацией химических веществ в батарее.Однако при протекании тока ЭДС отличается от выходного напряжения устройства. Напряжение на выводах батареи, например, меньше, чем ЭДС, когда батарея подает ток, и оно падает дальше, когда батарея разряжается или разряжается. Однако, если выходное напряжение устройства можно измерить без потребления тока, то выходное напряжение будет равно ЭДС (даже для сильно разряженной батареи).

Напряжение на клеммах

представляет схематическое изображение источника напряжения.Выходное напряжение устройства измеряется на его выводах и называется напряжением на выводах В . Напряжение на клеммах определяется уравнением:

Схематическое изображение источника напряжения : Любой источник напряжения (в данном случае углеродно-цинковый сухой элемент) имеет ЭДС, связанную с источником разности потенциалов, и внутреннее сопротивление r, связанное с его конструкцией. (Обратите внимание, что сценарий E означает ЭДС.) Также показаны выходные клеммы, на которых измеряется напряжение на клеммах V.Поскольку V = ЭДС-Ir, напряжение на клеммах равно ЭДС, только если ток не течет.

[латекс] \ text {V} = \ text {emf} — \ text {Ir} [/ latex],

где r — внутреннее сопротивление, а I — ток, протекающий во время измерения.

I является положительным, если ток течет от положительного вывода. Чем больше ток, тем меньше напряжение на клеммах. Точно так же верно, что чем больше внутреннее сопротивление, тем меньше напряжение на клеммах.

6.2 последовательных и параллельных резистора — Введение в электричество, магнетизм и схемы

ЦЕЛИ ОБУЧЕНИЯ

По окончании раздела вы сможете:
  • Определите термин эквивалентное сопротивление
  • Рассчитайте эквивалентное сопротивление резисторов, включенных последовательно
  • Рассчитайте эквивалентное сопротивление резисторов, включенных параллельно

В книге «Ток и сопротивление» мы описали термин «сопротивление» и объяснили основную конструкцию резистора.По сути, резистор ограничивает поток заряда в цепи и представляет собой омическое устройство, где. В большинстве схем имеется более одного резистора. Если несколько резисторов соединены вместе и подключены к батарее, ток, подаваемый батареей, зависит от эквивалентного сопротивления цепи.

Эквивалентное сопротивление комбинации резисторов зависит как от их индивидуальных значений, так и от способа их подключения. Простейшие комбинации резисторов — это последовательное и параллельное соединение (рисунок 6.2.1). В последовательной схеме выходной ток первого резистора течет на вход второго резистора; следовательно, ток одинаков в каждом резисторе. В параллельной схеме все выводы резистора на одной стороне резисторов соединены вместе, а все выводы на другой стороне соединены вместе. В случае параллельной конфигурации каждый резистор имеет одинаковое падение потенциала на нем, и токи через каждый резистор могут быть разными, в зависимости от резистора.Сумма отдельных токов равна току, протекающему по параллельным соединениям.

(Рисунок 6.2.1)

Рисунок 6.2.1 (a) При последовательном соединении резисторов ток на каждом резисторе одинаков. (b) При параллельном соединении резисторов напряжение на каждом резисторе одинаковое.

Резисторы серии

Считается, что резисторы

включены последовательно, когда ток течет через резисторы последовательно. Рассмотрим рисунок 6.2.2, на котором показаны три последовательно включенных резистора с приложенным напряжением, равным.Поскольку существует только один путь для прохождения зарядов, ток через каждый резистор одинаков. Эквивалентное сопротивление набора резисторов при последовательном соединении равно алгебраической сумме отдельных сопротивлений.

(Рисунок 6.2.2)

Рисунок 6.2.2 (a) Три резистора, подключенные последовательно к источнику напряжения. (b) Исходная схема сокращается до эквивалентного сопротивления и источника напряжения.

На рисунке 6.2.2 ток, идущий от источника напряжения, протекает через каждый резистор, поэтому ток через каждый резистор одинаков.Ток в цепи зависит от напряжения, подаваемого источником напряжения, и сопротивления резисторов. Для каждого резистора происходит падение потенциала, равное потере электрической потенциальной энергии при прохождении тока через каждый резистор. Согласно закону Ома падение потенциала на резисторе при протекании через него тока рассчитывается по формуле, где — ток в амперах (), а — сопротивление в омах (). Поскольку энергия сохраняется, а напряжение равно потенциальной энергии на заряд, сумма напряжения, приложенного к цепи источником, и падения потенциала на отдельных резисторах вокруг контура должны быть равны нулю:

Это уравнение часто называют законом петли Кирхгофа, который мы рассмотрим более подробно позже в этой главе.На рисунке 6.2.2 сумма падения потенциала каждого резистора и напряжения, подаваемого источником напряжения, должна равняться нулю:

Поскольку ток через каждый компонент одинаков, равенство можно упростить до эквивалентного сопротивления, которое представляет собой просто сумму сопротивлений отдельных резисторов.

Любое количество резисторов может быть подключено последовательно. Если резисторы соединены последовательно, эквивалентное сопротивление составляет

Ом.

(6.2.1)

Одним из результатов подключения компонентов в последовательную цепь является то, что если что-то происходит с одним компонентом, это влияет на все остальные компоненты. Например, если несколько ламп подключены последовательно и одна лампа перегорела, все остальные лампы погаснут.

ПРИМЕР 6.2.1


Эквивалентное сопротивление, ток и мощность в последовательной цепи

Батарея с напряжением на клеммах подключена к цепи, состоящей из четырех и одного резистора, соединенных последовательно (Рисунок 6.2.3). Предположим, что батарея имеет незначительное внутреннее сопротивление. (а) Рассчитайте эквивалентное сопротивление цепи. (б) Рассчитайте ток через каждый резистор. (c) Рассчитайте падение потенциала на каждом резисторе. (d) Определите общую мощность, рассеиваемую резисторами, и мощность, потребляемую батареей.

(Рисунок 6.2.3)

Рисунок 6.2.3 Простая последовательная схема с пятью резисторами.
Стратегия

В последовательной цепи эквивалентное сопротивление представляет собой алгебраическую сумму сопротивлений.Ток в цепи можно найти из закона Ома и равен напряжению, деленному на эквивалентное сопротивление. Падение потенциала на каждом резисторе можно найти с помощью закона Ома. Мощность, рассеиваемая каждым резистором, может быть найдена с помощью, а общая мощность, рассеиваемая резисторами, равна сумме мощности, рассеиваемой каждым резистором. Мощность, подаваемую аккумулятором, можно найти с помощью.

Решение

а. Эквивалентное сопротивление — это алгебраическая сумма сопротивлений: 90 · 109

г.Ток в цепи одинаков для каждого резистора в последовательной цепи и равен приложенному напряжению, деленному на эквивалентное сопротивление:

г. Падение потенциала на каждом резисторе можно найти с помощью закона Ома:

Обратите внимание, что сумма падений потенциала на каждом резисторе равна напряжению, подаваемому батареей.

г. Мощность, рассеиваемая резистором, равна, а мощность, отдаваемая аккумулятором, равна:

Значение

Существует несколько причин, по которым мы использовали бы несколько резисторов вместо одного резистора с сопротивлением, равным эквивалентному сопротивлению цепи.Возможно, резистора необходимого размера нет в наличии, или нам нужно отводить выделяемое тепло, или мы хотим минимизировать стоимость резисторов. Каждый резистор может стоить от нескольких центов до нескольких долларов, но при умножении на тысячи единиц экономия затрат может быть значительной.

ПРОВЕРЬТЕ ПОНИМАНИЕ 6.2

Некоторые гирлянды миниатюрных праздничных огней закорачиваются при перегорании лампочки. Устройство, вызывающее короткое замыкание, называется шунтом, который позволяет току течь по разомкнутой цепи.«Короткое замыкание» похоже на протягивание куска проволоки через компонент. Луковицы обычно сгруппированы в серию по девять луковиц. Если перегорает слишком много лампочек, в конечном итоге открываются шунты. Что вызывает это?

Кратко обозначим основные характеристики последовательно соединенных резисторов:

    Сопротивления серии
  1. суммируются, чтобы получить эквивалентное сопротивление:

  2. Один и тот же ток последовательно проходит через каждый резистор.
  3. Отдельные последовательно включенные резисторы не получают полное напряжение источника, а делят его.Общее падение потенциала на последовательной конфигурации резисторов равно сумме падений потенциала на каждом резисторе.

Параллельные резисторы

На рисунке 6.2.4 показаны резисторы, включенные параллельно, подключенные к источнику напряжения. Резисторы включены параллельно, когда один конец всех резисторов соединен непрерывным проводом с незначительным сопротивлением, а другой конец всех резисторов также соединен друг с другом непрерывным проводом с незначительным сопротивлением.Падение потенциала на каждом резисторе одинаковое. Ток через каждый резистор можно найти с помощью закона Ома, где напряжение на каждом резисторе постоянно. Например, автомобильные фары, радио и другие системы подключены параллельно, так что каждая подсистема использует полное напряжение источника и может работать полностью независимо. То же самое и с электропроводкой в ​​вашем доме или любом здании.

(Рисунок 6.2.4)

Рисунок 6.2.4 (a) Два резистора, подключенных параллельно источнику напряжения.(b) Исходная схема сокращается до эквивалентного сопротивления и источника напряжения.

Ток, протекающий от источника напряжения на рисунке 6.2.4, зависит от напряжения, подаваемого источником напряжения, и эквивалентного сопротивления цепи. В этом случае ток течет от источника напряжения и попадает в переход или узел, где цепь разделяется, протекая через резисторы и. По мере того, как заряды проходят от батареи, некоторые проходят через резистор, а некоторые — через резистор. Сумма токов, текущих в переход, должна быть равна сумме токов, текущих из перехода:

Это уравнение называется правилом соединения Кирхгофа и будет подробно обсуждено в следующем разделе.На рисунке 6.2.4 показано правило соединения. В этой схеме есть две петли, что приводит к уравнениям и. Обратите внимание, что напряжение на резисторах, включенных параллельно, одинаковое (), а ток является аддитивным:

Обобщая для любого количества резисторов, эквивалентное сопротивление параллельного соединения связано с отдельными сопротивлениями по формуле

(6.2.2)

Это соотношение приводит к эквивалентному сопротивлению, которое меньше наименьшего из отдельных сопротивлений.Когда резисторы подключены параллельно, от источника течет больше тока, чем протекает для любого из них по отдельности, поэтому общее сопротивление ниже.

ПРИМЕР 6.2.2


Анализ параллельной цепи

Три резистора, и подключены параллельно. Параллельное соединение подключается к источнику напряжения. а) Какое эквивалентное сопротивление? (b) Найдите ток, подаваемый источником в параллельную цепь. (c) Рассчитайте токи в каждом резисторе и покажите, что в сумме они равны выходному току источника.(d) Рассчитайте мощность, рассеиваемую каждым резистором. (e) Найдите выходную мощность источника и покажите, что она равна общей мощности, рассеиваемой резисторами.

Стратегия

(a) Общее сопротивление для параллельной комбинации резисторов находится с использованием.
(Обратите внимание, что в этих вычислениях каждый промежуточный ответ отображается с дополнительной цифрой.)

(b) Ток, подаваемый источником, можно найти по закону Ома, заменив полное сопротивление.

(c) Отдельные токи легко вычислить по закону Ома, поскольку каждый резистор получает полное напряжение.Полный ток — это сумма отдельных токов:.

(d) Мощность, рассеиваемая каждым резистором, может быть найдена с помощью любого из уравнений, связывающих мощность с током, напряжением и сопротивлением, поскольку все три известны. Давайте использовать, так как каждый резистор получает полное напряжение.

(e) Общую мощность также можно рассчитать несколькими способами, используйте.

Решение

а. Общее сопротивление для параллельной комбинации резисторов находится с помощью уравнения 6.2.2.Ввод известных значений дает

Общее сопротивление с правильным количеством значащих цифр составляет. Как и предполагалось, меньше минимального индивидуального сопротивления.

г. Полный ток можно найти из закона Ома, заменив полное сопротивление. Это дает

Ток для каждого устройства намного больше, чем для тех же устройств, подключенных последовательно (см. Предыдущий пример). Схема с параллельным соединением имеет меньшее общее сопротивление, чем резисторы, включенные последовательно.

г. Отдельные токи легко вычислить по закону Ома, поскольку на каждый резистор подается полное напряжение. Таким образом,

Аналогично

и

Полный ток складывается из отдельных токов:

г. Мощность, рассеиваемую каждым резистором, можно найти с помощью любого из уравнений, связывающих мощность с током, напряжением и сопротивлением, поскольку все три известны.Давайте использовать, так как каждый резистор получает полное напряжение. Таким образом,

Аналогично

и

e. Суммарную мощность также можно рассчитать несколькими способами. Выбор и ввод общей текущей доходности

Значение

Общая мощность, рассеиваемая резисторами, также составляет:

Обратите внимание, что общая мощность, рассеиваемая резисторами, равна мощности, подаваемой источником.

ПРОВЕРЬТЕ ПОНИМАНИЕ 6.3


Рассмотрим одну и ту же разность потенциалов, приложенную к одним и тем же трем последовательно включенным резисторам. Будет ли эквивалентное сопротивление последовательной цепи больше, меньше или равно трем резисторам, включенным параллельно? Будет ли ток в последовательной цепи выше, ниже или равен току, обеспечиваемому тем же напряжением, приложенным к параллельной цепи? Как мощность, рассеиваемая последовательно подключенными резисторами, будет сравниваться с мощностью, рассеиваемой параллельно резисторами?

ПРОВЕРЬТЕ ПОНИМАНИЕ 6.4


Как бы вы использовали реку и два водопада, чтобы смоделировать параллельную конфигурацию двух резисторов? Как разрушается эта аналогия?

Подведем итог основным характеристикам резисторов параллельно:

  1. Эквивалентное сопротивление находится из

    и меньше любого отдельного сопротивления в комбинации.

  2. Падение потенциала на каждом параллельном резисторе одинаковое.
  3. Параллельные резисторы не получают суммарный ток каждый; они делят это.Ток, поступающий в параллельную комбинацию резисторов, равен сумме токов, протекающих через каждый резистор, включенный параллельно.

В этой главе мы представили эквивалентное сопротивление резисторов, подключенных последовательно, и резисторов, подключенных параллельно. Вы можете вспомнить, что в разделе «Емкость» мы ввели эквивалентную емкость конденсаторов, соединенных последовательно и параллельно. Цепи часто содержат как конденсаторы, так и резисторы. В таблице 6.2.1 приведены уравнения, используемые для эквивалентного сопротивления и эквивалентной емкости для последовательных и параллельных соединений.

(таблица 6.2.1)

Комбинация серий Параллельная комбинация
Эквивалентная емкость
Эквивалентное сопротивление

Таблица 10.1 Сводка по эквивалентному сопротивлению и емкости в последовательной и параллельной комбинациях

Сочетания последовательного и параллельного

Более сложные соединения резисторов часто представляют собой просто комбинации последовательного и параллельного соединения.Такие комбинации обычны, особенно если учесть сопротивление проводов. В этом случае сопротивление провода включено последовательно с другими сопротивлениями, включенными параллельно.

Последовательное и параллельное соединение можно свести к одному эквивалентному сопротивлению, используя методику, показанную на рисунке 6.2.5. Различные части могут быть идентифицированы как последовательные или параллельные соединения, уменьшенные до их эквивалентных сопротивлений, а затем уменьшенные до тех пор, пока не останется единственное эквивалентное сопротивление. Процесс более трудоемкий, чем трудный.Здесь мы отмечаем эквивалентное сопротивление как.

(Рисунок 6.2.5)

Обратите внимание, что резисторы и включены последовательно. Их можно объединить в одно эквивалентное сопротивление. Один из методов отслеживания процесса — включить резисторы в качестве индексов. Здесь эквивалентное сопротивление и составляет

Теперь схема сокращается до трех резисторов, показанных на Рисунке 6.2.5 (c). Перерисовывая, мы теперь видим, что резисторы и составляют параллельную цепь.Эти два резистора можно уменьшить до эквивалентного сопротивления:

Этот шаг процесса сокращает схему до двух резисторов, показанных на Рисунке 6.2.5 (d). Здесь схема сводится к двум резисторам, которые в данном случае включены последовательно. Эти два резистора можно уменьшить до эквивалентного сопротивления, которое является эквивалентным сопротивлением цепи:

Основная цель этого анализа схемы достигнута, и теперь схема сведена к одному резистору и одному источнику напряжения.

Теперь мы можем проанализировать схему. Ток, обеспечиваемый источником напряжения, равен. Этот ток проходит через резистор и обозначен как. Падение потенциала можно найти с помощью закона Ома:

Глядя на рис. 6.2.5 (c), остается отбросить параллельную комбинацию и. Проходной ток можно найти с помощью закона Ома:

Резисторы и включены последовательно, поэтому токи и равны

Используя закон Ома, мы можем найти падение потенциала на двух последних резисторах.Потенциальные падения равны и. Окончательный анализ — это посмотреть на мощность, подаваемую источником напряжения, и мощность, рассеиваемую резисторами. Мощность, рассеиваемая на резисторах

Полная энергия постоянна в любом процессе. Следовательно, мощность, подаваемая источником напряжения, равна. Анализ мощности, подаваемой в схему, и мощности, рассеиваемой резисторами, является хорошей проверкой достоверности анализа; они должны быть равны.

ПРОВЕРЬТЕ ПОНИМАНИЕ 6.5


Рассмотрите электрические цепи в вашем доме. Приведите по крайней мере два примера схем, которые должны использовать комбинацию последовательных и параллельных схем для эффективной работы.

Практическое применение

Одним из следствий этого последнего примера является то, что сопротивление в проводах снижает ток и мощность, подаваемую на резистор. Если сопротивление провода относительно велико, как в изношенном (или очень длинном) удлинителе, то эти потери могут быть значительными. Если протекает большой ток, провал в проводах также может быть значительным и проявляться в виде тепла, выделяемого в шнуре.

Например, когда вы роетесь в холодильнике и включается мотор, свет холодильника на мгновение гаснет. Точно так же вы можете увидеть тусклый свет в салоне, когда вы запускаете двигатель вашего автомобиля (хотя это может быть связано с сопротивлением внутри самой батареи).

Что происходит в этих сильноточных ситуациях, показано на Рисунке 6.2.7. Устройство, представленное значком, имеет очень низкое сопротивление, поэтому при его включении протекает большой ток.Этот увеличенный ток вызывает большее падение в проводах, представленных значком, уменьшая напряжение на лампочке (которая есть), которое затем заметно гаснет.

(Рисунок 6.2.7)

Рисунок 6.2.7 Почему свет тускнеет, когда включен большой прибор? Ответ заключается в том, что большой ток, потребляемый двигателем прибора, вызывает значительное падение напряжения в проводах и снижает напряжение на свету.

Стратегия решения проблем: последовательные и параллельные резисторы


  1. Нарисуйте четкую принципиальную схему, пометив все резисторы и источники напряжения.Этот шаг включает список известных значений проблемы, поскольку они отмечены на вашей принципиальной схеме.
  2. Определите, что именно необходимо определить в проблеме (определите неизвестные). Письменный список полезен.
  3. Определите, включены ли резисторы последовательно, параллельно или в комбинации последовательно и параллельно. Изучите принципиальную схему, чтобы сделать эту оценку. Резисторы включены последовательно, если через них должен последовательно проходить один и тот же ток.
  4. Используйте соответствующий список основных функций для последовательных или параллельных подключений, чтобы найти неизвестные.Есть один список для серий, а другой — для параллелей.
  5. Проверьте, являются ли ответы разумными и последовательными.

ПРИМЕР 6.2.4


Объединение последовательных и параллельных цепей

Два резистора, соединенных последовательно, подключены к двум резисторам, включенным параллельно. Последовательно-параллельная комбинация подключается к батарее. Каждый резистор имеет сопротивление. Провода, соединяющие резисторы и аккумулятор, имеют незначительное сопротивление.Ток проходит через резистор. Какое напряжение подается от источника напряжения?

Стратегия

Используйте шаги предыдущей стратегии решения проблем, чтобы найти решение для этого примера.

Решение
  1. Нарисуйте четкую принципиальную схему (рисунок 6.2.8).

    (Рисунок 6.2.8)

    Рисунок 6.2.8 Чтобы найти неизвестное напряжение, мы должны сначала найти эквивалентное сопротивление цепи.
  2. Неизвестно напряжение аккумулятора.Чтобы определить напряжение, подаваемое батареей, необходимо найти эквивалентное сопротивление.
  3. В этой схеме мы уже знаем, что резисторы и включены последовательно, а резисторы и включены параллельно. Эквивалентное сопротивление параллельной конфигурации резисторов и последовательно с последовательной конфигурацией резисторов и.
  4. Напряжение, подаваемое батареей, можно найти, умножив ток от батареи на эквивалентное сопротивление цепи.Ток от аккумулятора равен току через него и равен. Нам нужно найти эквивалентное сопротивление, уменьшив схему. Чтобы уменьшить схему, сначала рассмотрите два резистора, включенных параллельно. Эквивалентное сопротивление составляет. Эта параллельная комбинация включена последовательно с двумя другими резисторами, поэтому эквивалентное сопротивление цепи равно. Таким образом, напряжение, подаваемое батареей, составляет.
  5. Один из способов проверить соответствие ваших результатов — это рассчитать мощность, подаваемую батареей, и мощность, рассеиваемую резисторами.Мощность, подаваемая аккумулятором, составляет

    Поскольку они включены последовательно, сквозной ток равен сквозному току. Т.к. ток через каждый будет. Мощность, рассеиваемая резисторами, равна сумме мощности, рассеиваемой каждым резистором:


    Поскольку мощность, рассеиваемая резисторами, равна мощности, выделяемой батареей, наше решение кажется последовательным.

Значение

Если проблема имеет комбинацию последовательного и параллельного соединения, как в этом примере, ее можно уменьшить поэтапно, используя предыдущую стратегию решения проблемы и рассматривая отдельные группы последовательных или параллельных соединений.При поиске параллельного подключения необходимо соблюдать осторожность. Кроме того, единицы и числовые результаты должны быть разумными. Эквивалентное последовательное сопротивление должно быть больше, а эквивалентное параллельное сопротивление, например, должно быть меньше. Мощность должна быть больше для одних и тех же устройств, подключенных параллельно, по сравнению с последовательными и т. Д.

Кандела Цитаты

Лицензионный контент CC, конкретная атрибуция

  • Загрузите бесплатно по адресу http: // cnx.org/contents/[email protected]. Получено с : http://cnx.org/contents/[email protected]. Лицензия : CC BY: Attribution

10.3: Последовательные и параллельные резисторы

Цели обучения

К концу раздела вы сможете:

  • Определите термин эквивалентное сопротивление
  • Рассчитайте эквивалентное сопротивление резисторов, включенных последовательно
  • Рассчитайте эквивалентное сопротивление резисторов, включенных параллельно

В книге «Ток и сопротивление» мы описали термин «сопротивление» и объяснили основную конструкцию резистора.По сути, резистор ограничивает поток заряда в цепи и представляет собой омическое устройство, где \ (V = IR \). В большинстве схем имеется более одного резистора. Если несколько резисторов соединены вместе и подключены к батарее, ток, подаваемый батареей, зависит от эквивалентного сопротивления цепи.

Эквивалентное сопротивление комбинации резисторов зависит как от их индивидуальных значений, так и от способа их подключения. Самыми простыми комбинациями резисторов являются последовательное и параллельное соединение (Рисунок \ (\ PageIndex {1} \)).В последовательной схеме выходной ток первого резистора течет на вход второго резистора; следовательно, ток одинаков в каждом резисторе. В параллельной схеме все выводы резистора на одной стороне резисторов соединены вместе, а все выводы на другой стороне соединены вместе. В случае параллельной конфигурации каждый резистор имеет одинаковое падение потенциала на нем, и токи через каждый резистор могут быть разными, в зависимости от резистора.Сумма отдельных токов равна току, протекающему по параллельным соединениям.

Рисунок \ (\ PageIndex {1} \): (a) При последовательном соединении резисторов ток одинаков в каждом резисторе. (b) При параллельном соединении резисторов напряжение на каждом резисторе одинаковое.

Резисторы серии

Считается, что резисторы

включены последовательно, когда ток течет через резисторы последовательно. Рассмотрим рисунок \ (\ PageIndex {2} \), на котором показаны три последовательно включенных резистора с приложенным напряжением, равным \ (V_ {ab} \).Поскольку существует только один путь для прохождения зарядов, ток через каждый резистор одинаков. Эквивалентное сопротивление набора резисторов при последовательном соединении равно алгебраической сумме отдельных сопротивлений.

Рисунок \ (\ PageIndex {2} \): (a) Три резистора, подключенные последовательно к источнику напряжения. (b) Исходная схема сокращается до эквивалентного сопротивления и источника напряжения.

На рисунке \ (\ PageIndex {2} \) ток, идущий от источника напряжения, протекает через каждый резистор, поэтому ток через каждый резистор одинаков.Ток в цепи зависит от напряжения, подаваемого источником напряжения, и сопротивления резисторов. Для каждого резистора происходит падение потенциала, равное потере электрической потенциальной энергии при прохождении тока через каждый резистор. Согласно закону Ома, падение потенциала \ (V \) на резисторе при протекании через него тока рассчитывается по формуле \ (V = IR \), где \ (I \) — ток в амперах (\ (A \)), а \ (R \) — сопротивление в Ом \ ((\ Omega) \).N V_i = 0. \]

Это уравнение часто называют законом петли Кирхгофа, который мы рассмотрим более подробно позже в этой главе. На рисунке \ (\ PageIndex {2} \) сумма падения потенциала каждого резистора и напряжения, подаваемого источником напряжения, должна равняться нулю:

\ [\ begin {align *} V — V_1 — V_2 — V_3 & = 0, \\ [4pt] V & = V_1 + V_2 + V_3, \\ [4pt] & = IR_1 + IR_2 + IR_3, \ end { выровнять *} \]

Решение для \ (I \)

\ [\ begin {align *} I & = \ frac {V} {R_1 + R_2 + R_3} \\ [4pt] & = \ frac {V} {R_ {S}}.\ end {align *} \]

Поскольку ток через каждый компонент одинаков, равенство можно упростить до эквивалентного сопротивления (\ (R_ {S} \)), которое представляет собой просто сумму сопротивлений отдельных резисторов.

Эквивалентное сопротивление в последовательной цепи

Любое количество резисторов может быть подключено последовательно. Если \ (N \) резисторы соединены последовательно, эквивалентное сопротивление равно

. N R_i.\ label {серия эквивалентных сопротивлений} \]

Одним из результатов подключения компонентов в последовательную цепь является то, что если что-то происходит с одним компонентом, это влияет на все остальные компоненты. Например, если несколько ламп подключены последовательно и одна лампа перегорела, все остальные лампы погаснут.

Пример \ (\ PageIndex {1} \): эквивалентное сопротивление, ток и мощность в последовательной цепи

Батарея с напряжением на клеммах 9 В подключена к цепи, состоящей из четырех последовательно соединенных резисторов \ (20 \, \ Omega \) и одного \ (10 ​​\, \ Omega \) (Рисунок \ (\ PageIndex {3 } \)).Предположим, что батарея имеет незначительное внутреннее сопротивление.

  1. Рассчитайте эквивалентное сопротивление цепи.
  2. Рассчитайте ток через каждый резистор.
  3. Рассчитайте падение потенциала на каждом резисторе.
  4. Определите общую мощность, рассеиваемую резисторами, и мощность, потребляемую батареей.
Рисунок \ (\ PageIndex {3} \): Простая последовательная схема с пятью резисторами.

Стратегия

В последовательной цепи эквивалентное сопротивление представляет собой алгебраическую сумму сопротивлений.2R \), а общая мощность, рассеиваемая резисторами, равна сумме мощности, рассеиваемой каждым резистором. Мощность, подаваемая батареей, можно найти с помощью \ (P = I \ epsilon \).

Решение

  1. Эквивалентное сопротивление — это алгебраическая сумма сопротивлений (Уравнение \ ref {серия эквивалентных сопротивлений}): \ [\ begin {align *} R_ {S} & = R_1 + R_2 + R_3 + R_4 + R_5 \\ [4pt ] & = 20 \, \ Омега + 20 \, \ Омега + 20 \, \ Омега + 20 \, \ Омега + 10 \, \ Омега = 90 \, \ Омега.2 (10 \, \ Omega) = 0,1 \, W, \ nonumber \] \ [P_ {рассеивается} = 0,2 \, W + 0,2 \, W + 0,2 \, W + 0,2 \, W + 0,1 \, W = 0,9 \, W, \ nonumber \] \ [P_ {источник} = I \ epsilon = (0,1 \, A) (9 \, V) = 0,9 \, W. \ nonumber \]

Значение

Существует несколько причин, по которым мы использовали бы несколько резисторов вместо одного резистора с сопротивлением, равным эквивалентному сопротивлению цепи. Возможно, резистора необходимого размера нет в наличии, или нам нужно отводить выделяемое тепло, или мы хотим минимизировать стоимость резисторов.Каждый резистор может стоить от нескольких центов до нескольких долларов, но при умножении на тысячи единиц экономия затрат может быть значительной.

Упражнение \ (\ PageIndex {1} \)

Некоторые гирлянды миниатюрных праздничных огней закорачиваются при перегорании лампочки. Устройство, вызывающее короткое замыкание, называется шунтом, который позволяет току течь по разомкнутой цепи. «Короткое замыкание» похоже на протягивание куска проволоки через компонент. Луковицы обычно сгруппированы в серию по девять луковиц.Если перегорает слишком много лампочек, в конечном итоге открываются шунты. Что вызывает это?

Ответ

Эквивалентное сопротивление девяти последовательно соединенных лампочек составляет 9 R . Ток равен \ (I = V / 9 \, R \). Если одна лампочка перегорит, эквивалентное сопротивление составит 8 R , и напряжение не изменится, но ток возрастет \ ((I = V / 8 \, R \). Чем больше лампочек перегорят, ток станет равным. В конце концов, ток становится слишком большим, что приводит к сгоранию шунта.№ Р_и. \]

  • Один и тот же ток последовательно проходит через каждый резистор.
  • Отдельные последовательно включенные резисторы не получают полное напряжение источника, а делят его. Общее падение потенциала на последовательной конфигурации резисторов равно сумме падений потенциала на каждом резисторе.
  • Параллельные резисторы

    На рисунке \ (\ PageIndex {4} \) показаны резисторы, включенные параллельно, подключенные к источнику напряжения. Резисторы включены параллельно, когда один конец всех резисторов соединен непрерывным проводом с незначительным сопротивлением, а другой конец всех резисторов также соединен друг с другом непрерывным проводом с незначительным сопротивлением.Падение потенциала на каждом резисторе одинаковое. Ток через каждый резистор можно найти с помощью закона Ома \ (I = V / R \), где напряжение на каждом резисторе постоянно. Например, автомобильные фары, радио и другие системы подключены параллельно, так что каждая подсистема использует полное напряжение источника и может работать полностью независимо. То же самое и с электропроводкой в ​​вашем доме или любом здании.

    Рисунок \ (\ PageIndex {4} \): Два резистора, подключенных параллельно источнику напряжения.(b) Исходная схема сокращается до эквивалентного сопротивления и источника напряжения.

    Ток, протекающий от источника напряжения на рисунке \ (\ PageIndex {4} \), зависит от напряжения, подаваемого источником напряжения, и эквивалентного сопротивления цепи. В этом случае ток течет от источника напряжения и попадает в переход или узел, где цепь разделяется, протекая через резисторы \ (R_1 \) и \ (R_2 \). По мере того, как заряды идут от аккумулятора, некоторые проходят через резистор \ (R_1 \), а некоторые — через резистор \ (R_2 \).Сумма токов, текущих в переход, должна быть равна сумме токов, текущих из перехода:

    \ [\ sum I_ {in} = \ sum I_ {out}. {- 1}.{-1}. \ label {10.3} \]

    Это соотношение приводит к эквивалентному сопротивлению \ (R_ {P} \), которое меньше наименьшего из отдельных сопротивлений. Когда резисторы подключены параллельно, от источника течет больше тока, чем протекает для любого из них по отдельности, поэтому общее сопротивление ниже.

    Пример \ (\ PageIndex {2} \): Анализ параллельной цепи

    Три резистора \ (R_1 = 1,00 \, \ Omega \), \ (R_2 = 2,00 \, \ Omega \) и \ (R_3 = 2,00 \, \ Omega \) подключены параллельно.Параллельное соединение подключается к источнику напряжения \ (V = 3,00 \, V \).

    1. Какое эквивалентное сопротивление?
    2. Найдите ток, подаваемый источником в параллельную цепь.
    3. Рассчитайте токи в каждом резисторе и покажите, что в сумме они равны выходному току источника.
    4. Рассчитайте мощность, рассеиваемую каждым резистором.
    5. Найдите выходную мощность источника и покажите, что она равна общей мощности, рассеиваемой резисторами.

    Стратегия

    (a) Общее сопротивление для параллельной комбинации резисторов находится с помощью уравнения \ ref {10.3}. (Обратите внимание, что в этих вычислениях каждый промежуточный ответ отображается с дополнительной цифрой.)

    (b) Ток, подаваемый источником, можно найти из закона Ома, заменив \ (R_ {P} \) на полное сопротивление \ (I = \ frac {V} {R_ {P}} \).

    (c) Отдельные токи легко вычислить по закону Ома \ (\ left (I_i = \ frac {V_i} {R_i} \ right) \), поскольку каждый резистор получает полное напряжение.{-1} = 0,50 \, \ Omega. \ Nonumber \] Общее сопротивление с правильным количеством значащих цифр равно \ (R_ {eq} = 0,50 \, \ Omega \). Как и предполагалось, \ (R_ {P} \) меньше наименьшего индивидуального сопротивления.

  • Полный ток можно найти из закона Ома, заменив полное сопротивление \ (R_ {P} \). Это дает \ [I = \ frac {V} {R_ {P}} = \ frac {3.00 \, V} {0.50 \, \ Omega} = 6.00 \, A. \ nonumber \] Текущий I для каждого устройства намного больше, чем для тех же устройств, подключенных последовательно (см. предыдущий пример).Схема с параллельным соединением имеет меньшее общее сопротивление, чем резисторы, включенные последовательно.
  • Отдельные токи легко вычислить по закону Ома, поскольку каждый резистор получает полное напряжение. Таким образом, \ [I_1 = \ frac {V} {R_1} = \ frac {3.00 \, V} {1.00 \, \ Omega} = 3.00 \, A. \ nonumber \] Аналогично, \ [I_2 = \ frac {V } {R_2} = \ frac {3.00 \, V} {2.00 \, \ Omega} = 1.50 \, A \ nonumber \] и \ [I_3 = \ frac {V} {R_3} = \ frac {3.00 \, V } {2.00 \, \ Omega} = 1.50 \, A. \ nonumber \] Полный ток — это сумма отдельных токов: \ [I_1 + I_2 + I_3 = 6.2} {2.00 \, \ Omega} = 4.50 \, W. \ nonumber \]
  • Общую мощность также можно рассчитать несколькими способами. Выбор \ (P = IV \) и ввод общей текущей доходности \ [P = IV = (6.00 \, A) (3.00 \, V) = 18.00 \, W. \ nonumber \]
  • Значение

    Общая мощность, рассеиваемая резисторами, также составляет 18,00 Вт:

    \ [P_1 + P_2 + P_3 = 9,00 \, W + 4,50 \, W + 4,50 \, W = 18,00 \, W. \ nonumber \]

    Обратите внимание, что общая мощность, рассеиваемая резисторами, равна мощности, подаваемой источником.

    Упражнение \ (\ PageIndex {2A} \)

    Рассмотрим ту же самую разность потенциалов \ ((V = 3,00 \, V) \), приложенную к одним и тем же трем последовательно включенным резисторам. Будет ли эквивалентное сопротивление последовательной цепи больше, меньше или равно трем резисторам, включенным параллельно? Будет ли ток в последовательной цепи выше, ниже или равен току, обеспечиваемому тем же напряжением, приложенным к параллельной цепи? Как мощность, рассеиваемая последовательно подключенными резисторами, будет сравниваться с мощностью, рассеиваемой параллельно резисторами?

    Решение

    Эквивалент последовательной схемы будет \ (R_ {eq} = 1.00 \, \ Omega + 2.00 \, \ Omega + 2.00 \, \ Omega = 5.00 \, \ Omega \), что выше эквивалентного сопротивления параллельной цепи \ (R_ {eq} = 0.50 \, \ Omega \ ). Эквивалентное сопротивление любого количества резисторов всегда выше, чем эквивалентное сопротивление тех же резисторов, соединенных параллельно. Ток через последовательную цепь будет равен \ (I = \ frac {3.00 \, V} {5.00 \, \ Omega} = 0.60 \, A \), что меньше суммы токов, проходящих через каждый резистор в параллельная цепь, \ (I = 6.00 \, А \). Это неудивительно, поскольку эквивалентное сопротивление последовательной цепи выше. Ток при последовательном соединении любого количества резисторов всегда будет ниже, чем ток при параллельном соединении тех же резисторов, поскольку эквивалентное сопротивление последовательной цепи будет выше, чем параллельной цепи. Мощность, рассеиваемая последовательно подключенными резисторами, будет равна \ (P = 1,800 \, Вт \), что ниже мощности, рассеиваемой в параллельной цепи \ (P = 18.00 \, Вт \).

    Упражнение \ (\ PageIndex {2B} \)

    Как бы вы использовали реку и два водопада, чтобы смоделировать параллельную конфигурацию двух резисторов? Как разрушается эта аналогия?

    Решение

    Река, текущая горизонтально с постоянной скоростью, разделяется на две части и течет через два водопада. Молекулы воды аналогичны электронам в параллельных цепях. Количество молекул воды, которые текут в реке и падает, должно быть равно количеству молекул, которые текут над каждым водопадом, точно так же, как сумма тока через каждый резистор должна быть равна току, текущему в параллельном контуре.Молекулы воды в реке обладают энергией благодаря своему движению и высоте. Потенциальная энергия молекул воды в реке постоянна из-за их одинаковой высоты. Это аналогично постоянному изменению напряжения в параллельной цепи. Напряжение — это потенциальная энергия на каждом резисторе.

    При рассмотрении энергии аналогия быстро разрушается. В водопаде потенциальная энергия преобразуется в кинетическую энергию молекул воды. В случае прохождения электронов через резистор падение потенциала преобразуется в тепло и свет, а не в кинетическую энергию электронов.

    Подведем итог основным характеристикам резисторов параллельно:

    1. Эквивалентное сопротивление находится по формуле \ ref {10.3} и меньше любого отдельного сопротивления в комбинации.
    2. Падение потенциала на каждом параллельном резисторе одинаковое.
    3. Параллельные резисторы не получают суммарный ток каждый; они делят это. Ток, поступающий в параллельную комбинацию резисторов, равен сумме токов, протекающих через каждый резистор, включенный параллельно.

    В этой главе мы представили эквивалентное сопротивление резисторов, подключенных последовательно, и резисторов, подключенных параллельно. Как вы помните, в разделе «Емкость» мы ввели эквивалентную емкость конденсаторов, соединенных последовательно и параллельно. Цепи часто содержат как конденсаторы, так и резисторы. Таблица \ (\ PageIndex {1} \) суммирует уравнения, используемые для эквивалентного сопротивления и эквивалентной емкости для последовательных и параллельных соединений.

    Таблица \ (\ PageIndex {1} \): Сводка по эквивалентному сопротивлению и емкости в последовательной и параллельной комбинациях
    Комбинация серий Параллельная комбинация
    Эквивалентная емкость \ [\ frac {1} {C_ {S}} = \ frac {1} {C_1} + \ frac {1} {C_2} + \ frac {1} {C_3} +.N R_i \ nonumber \] \ [\ frac {1} {R_ {P}} = \ frac {1} {R_1} + \ frac {1} {R_2} + \ frac {1} {R_3} +. . . \ nonumber \]

    Сочетания последовательного и параллельного

    Более сложные соединения резисторов часто представляют собой просто комбинации последовательного и параллельного соединения. Такие комбинации обычны, особенно если учесть сопротивление проводов. В этом случае сопротивление провода включено последовательно с другими сопротивлениями, включенными параллельно.

    Комбинации последовательного и параллельного соединения можно свести к одному эквивалентному сопротивлению, используя технику, показанную на рисунке \ (\ PageIndex {5} \).Различные части могут быть идентифицированы как последовательные или параллельные соединения, уменьшенные до их эквивалентных сопротивлений, а затем уменьшенные до тех пор, пока не останется единственное эквивалентное сопротивление. Процесс более трудоемкий, чем трудный. Здесь мы отмечаем эквивалентное сопротивление как \ (R_ {eq} \).

    Рисунок \ (\ PageIndex {5} \): (а) Исходная схема из четырех резисторов. (b) Шаг 1: резисторы \ (R_3 \) и \ (R_4 \) включены последовательно, и эквивалентное сопротивление равно \ (R_ {34} = 10 \, \ Omega \). (c) Шаг 2: сокращенная схема показывает, что резисторы \ (R_2 \) и \ (R_ {34} \) включены параллельно, с эквивалентным сопротивлением \ (R_ {234} = 5 \, \ Omega \).(d) Шаг 3: сокращенная схема показывает, что \ (R_1 \) и \ (R_ {234} \) включены последовательно с эквивалентным сопротивлением \ (R_ {1234} = 12 \, \ Omega \), которое является эквивалентное сопротивление \ (R_ {eq} \). (e) Уменьшенная схема с источником напряжения \ (V = 24 \, V \) с эквивалентным сопротивлением \ (R_ {eq} = 12 \, \ Omega \). Это приводит к току \ (I = 2 \, A \) от источника напряжения.

    Обратите внимание, что резисторы \ (R_3 \) и \ (R_4 \) включены последовательно. Их можно объединить в одно эквивалентное сопротивление.Один из методов отслеживания процесса — включить резисторы в качестве индексов. {- 1} = 5 \, \ Omega.\ nonumber \]

    Этот шаг процесса сокращает схему до двух резисторов, показанных на рисунке \ (\ PageIndex {5d} \). Здесь схема сводится к двум резисторам, которые в данном случае включены последовательно. Эти два резистора можно уменьшить до эквивалентного сопротивления, которое является эквивалентным сопротивлением цепи:

    \ [R_ {eq} = R_ {1234} = R_1 + R_ {234} = 7 \, \ Omega + 5 \ Omega = 12 \, \ Omega. \ nonumber \]

    Основная цель этого анализа схемы достигнута, и теперь схема сведена к одному резистору и одному источнику напряжения.

    Теперь мы можем проанализировать схему. Ток, обеспечиваемый источником напряжения, равен \ (I = \ frac {V} {R_ {eq}} = \ frac {24 \, V} {12 \, \ Omega} = 2 \, A \). Этот ток проходит через резистор \ (R_1 \) и обозначается как \ (I_1 \). Падение потенциала на \ (R_1 \) можно найти с помощью закона Ома:

    \ [V_1 = I_1R_1 = (2 \, A) (7 \, \ Omega) = 14 \, V. \ nonumber \]

    Глядя на рисунок \ (\ PageIndex {5c} \), это оставляет \ (24 \, V — 14 \, V = 10 \, V \), чтобы отбросить его через параллельную комбинацию \ (R_2 \) и \ ( R_ {34} \).Ток через \ (R_2 \) можно найти по закону Ома:

    \ [I_2 = \ frac {V_2} {R_2} = \ frac {10 \, V} {10 \, \ Omega} = 1 \, A. \ nonumber \]

    Резисторы \ (R_3 \) и \ (R_4 \) включены последовательно, поэтому токи \ (I_3 \) и \ (I_4 \) равны

    .

    \ [I_3 = I_4 = I — I_2 = 2 \, A — 1 \, A = 1 \, A. \ nonumber \]

    Используя закон Ома, мы можем найти падение потенциала на двух последних резисторах. Потенциальные капли равны \ (V_3 = I_3R_3 = 6 \, V \) и \ (V_4 = I_4R_4 = 4 \, V \).2 (4 \, \ Omega) = 4 \, W, \\ [4pt] P_ {рассеивается} & = P_1 + P_2 + P_3 + P_4 = 48 \, W. \ end {align *} \]

    Полная энергия постоянна в любом процессе. Следовательно, мощность, подаваемая источником напряжения, составляет

    \ [\ begin {align *} P_s & = IV \\ [4pt] & = (2 \, A) (24 \, V) = 48 \, W \ end {align *} \]

    Анализ мощности, подаваемой в схему, и мощности, рассеиваемой резисторами, является хорошей проверкой достоверности анализа; они должны быть равны.

    Пример \ (\ PageIndex {3} \): объединение последовательных и параллельных цепей

    На рисунке \ (\ PageIndex {6} \) показаны резисторы, подключенные последовательно и параллельно.Мы можем считать \ (R_1 \) сопротивлением проводов, ведущих к \ (R_2 \) и \ (R_3 \).

    1. Найдите эквивалентное сопротивление цепи.
    2. Какое падение потенциала \ (V_1 \) на резисторе \ (R_1 \)?
    3. Найдите ток \ (I_2 \) через резистор \ (R_2 \).
    4. Какая мощность рассеивается \ (R_2 \)?
    Рисунок \ (\ PageIndex {6} \): Эти три резистора подключены к источнику напряжения так, чтобы \ (R_2 \) и \ (R_3 \) были параллельны друг другу, и эта комбинация была последовательно с \ (R_1 \).

    Стратегия

    (a) Чтобы найти эквивалентное сопротивление, сначала найдите эквивалентное сопротивление параллельного соединения \ (R_2 \) и \ (R_3 \). Затем используйте этот результат, чтобы найти эквивалентное сопротивление последовательного соединения с \ (R_1 \).

    (b) Ток через \ (R_1 \) можно найти с помощью закона Ома и приложенного напряжения. Ток через \ (R_1 \) равен току от батареи. Падение потенциала \ (V_1 \) на резисторе \ (R_1 \) (которое представляет собой сопротивление в соединительных проводах) можно найти с помощью закона Ома.{-1} = 5.10 \, \ Omega. \ Nonumber \] Общее сопротивление этой комбинации является промежуточным между значениями чистой серии и чисто параллельной (\ (20.0 \, \ Omega \) и \ (0.804 \, \ Omega \) ), соответственно).

  • Ток через \ (R_1 \) равен току, обеспечиваемому батареей: \ [I_1 = I = \ frac {V} {R_ {eq}} = \ frac {12.0 \, V} {5.10 \, \ Omega} = 2.35 \, A. \ nonumber \] Напряжение на \ (R_1 \) равно \ [V_1 = I_1R_1 = (2.35 \, A) (1 \, \ Omega) = 2.35 \, V. \ nonumber \] Напряжение, приложенное к \ (R_2 \) и \ (R_3 \), меньше напряжения, подаваемого батареей, на величину \ (V_1 \).Когда сопротивление провода велико, это может существенно повлиять на работу устройств, представленных \ (R_2 \) и \ (R_3 \).
  • Чтобы найти ток через \ (R_2 \), мы должны сначала найти приложенное к нему напряжение. Напряжение на двух параллельных резисторах одинаковое: \ [V_2 = V_3 = V — V_1 = 12.0 \, V — 2.35 \, V = 9.65 \, V. \ nonumber \] Теперь мы можем найти ток \ (I_2 \) через сопротивление \ (R_2 \) по закону Ома: \ [I_2 = \ frac {V_2} {R_2} = \ frac {9.65 \, V} {6.00 \, \ Omega} = 1.2 (6.00 \, \ Omega) = 15.5 \, W. \ nonumber \]
  • Значение

    Анализ сложных схем часто можно упростить, сведя схему к источнику напряжения и эквивалентному сопротивлению. Даже если вся схема не может быть сведена к одному источнику напряжения и одному эквивалентному сопротивлению, части схемы могут быть уменьшены, что значительно упрощает анализ.

    Упражнение \ (\ PageIndex {3} \)

    Рассмотрите электрические цепи в вашем доме.Приведите по крайней мере два примера схем, которые должны использовать комбинацию последовательных и параллельных схем для эффективной работы.

    Решение

    Все цепи верхнего освещения параллельны и подключены к основному питанию, поэтому при перегорании одной лампочки все верхнее освещение не гаснет. У каждого верхнего света будет по крайней мере один переключатель, включенный последовательно с источником света, поэтому вы можете включать и выключать его.

    В холодильнике есть компрессор и лампа, которая загорается при открытии дверцы.Обычно у холодильника есть только один шнур для подключения к стене. Цепь, содержащая компрессор, и цепь, содержащая цепь освещения, параллельны, но есть переключатель, включенный последовательно со светом. Термостат управляет переключателем, который включен последовательно с компрессором, чтобы контролировать температуру холодильника.

    Практическое применение

    Одним из следствий этого последнего примера является то, что сопротивление в проводах снижает ток и мощность, подаваемую на резистор.Если сопротивление провода относительно велико, как в изношенном (или очень длинном) удлинителе, то эти потери могут быть значительными. Если потребляется большой ток, падение IR в проводах также может быть значительным и может проявляться из-за тепла, выделяемого в шнуре.

    Например, когда вы роетесь в холодильнике и включается мотор, свет холодильника на мгновение гаснет. Точно так же вы можете увидеть тусклый свет в салоне, когда вы запускаете двигатель вашего автомобиля (хотя это может быть связано с сопротивлением внутри самой батареи).

    Что происходит в этих сильноточных ситуациях, показано на рисунке \ (\ PageIndex {7} \). Устройство, обозначенное символом \ (R_3 \), имеет очень низкое сопротивление, поэтому при его включении протекает большой ток. Этот увеличенный ток вызывает большее падение IR в проводах, обозначенных \ (R_1 \), снижая напряжение на лампочке (которое равно \ (R_2 \)), которое затем заметно гаснет.

    Рисунок \ (\ PageIndex {7} \): Почему свет тускнеет, когда включен большой прибор? Ответ заключается в том, что большой ток, потребляемый двигателем прибора, вызывает значительное падение IR в проводах и снижает напряжение на свету.

    Стратегия решения проблем: последовательные и параллельные резисторы

    1. Нарисуйте четкую принципиальную схему, пометив все резисторы и источники напряжения. Этот шаг включает список известных значений проблемы, поскольку они отмечены на вашей принципиальной схеме.
    2. Определите, что именно необходимо определить в проблеме (определите неизвестные). Письменный список полезен.
    3. Определите, включены ли резисторы последовательно, параллельно или в комбинации последовательно и параллельно.Изучите принципиальную схему, чтобы сделать эту оценку. Резисторы включены последовательно, если через них должен последовательно проходить один и тот же ток.
    4. Используйте соответствующий список основных функций для последовательных или параллельных подключений, чтобы найти неизвестные. Есть один список для серий, а другой — для параллелей.
    5. Проверьте, являются ли ответы разумными и последовательными.

    Пример \ (\ PageIndex {4} \): объединение последовательных и параллельных цепей

    Два резистора, соединенных последовательно \ ((R_1, \, R_2) \), подключены к двум резисторам, включенным параллельно \ ((R_3, \, R_4) \).Последовательно-параллельная комбинация подключается к батарее. Каждый резистор имеет сопротивление 10,00 Ом. Провода, соединяющие резисторы и аккумулятор, имеют незначительное сопротивление. Через резистор \ (R_1 \) проходит ток 2,00 А. Какое напряжение подается от источника напряжения?

    Стратегия

    Используйте шаги предыдущей стратегии решения проблем, чтобы найти решение для этого примера.

    Решение

    Рисунок \ (\ PageIndex {8} \): Чтобы найти неизвестное напряжение, мы должны сначала найти эквивалентное сопротивление цепи.
    1. Нарисуйте четкую принципиальную схему (рисунок \ (\ PageIndex {8} \)).
    2. Неизвестно напряжение аккумулятора. Чтобы определить напряжение, подаваемое батареей, необходимо найти эквивалентное сопротивление.
    3. В этой схеме мы уже знаем, что резисторы \ (R_1 \) и \ (R_2 \) включены последовательно, а резисторы \ (R_3 \) и \ (R_4 \) включены параллельно. Эквивалентное сопротивление параллельной конфигурации резисторов \ (R_3 \) и \ (R_4 \) последовательно с последовательной конфигурацией резисторов \ (R_1 \) и \ (R_2 \).{-1} = 5,00 \, \ Омега. \ nonumber \] Эта параллельная комбинация включена последовательно с двумя другими резисторами, поэтому эквивалентное сопротивление схемы равно \ (R_ {eq} = R_1 + R_2 + R_ {34} = (25.00 \, \ Omega \). поэтому напряжение, подаваемое батареей, равно \ (V = IR_ {eq} = 2.00 \, A (25.00 \, \ Omega) = 50.00 \, V \).
    4. Один из способов проверить соответствие ваших результатов — это рассчитать мощность, подаваемую батареей, и мощность, рассеиваемую резисторами. Мощность, обеспечиваемая аккумулятором, равна \ (P_ {batt} = IV = 100.2R_4 \\ [4pt] & = 40.00 \, W + 40.00 \, W + 10.00 \, W + 10.00 \, W = 100. \, W. \ end {align *} \]

      Поскольку мощность, рассеиваемая резисторами, равна мощности, выделяемой батареей, наше решение кажется последовательным.

      Значение

      Если проблема имеет комбинацию последовательного и параллельного соединения, как в этом примере, ее можно уменьшить поэтапно, используя предыдущую стратегию решения проблемы и рассматривая отдельные группы последовательных или параллельных соединений.При нахождении \ (R_ {eq} \) для параллельного соединения необходимо с осторожностью относиться к обратному. Кроме того, единицы и числовые результаты должны быть разумными. Эквивалентное последовательное сопротивление должно быть больше, а эквивалентное параллельное сопротивление, например, должно быть меньше. Мощность должна быть больше для одних и тех же устройств, подключенных параллельно, по сравнению с последовательными и т. Д.

      Авторы и авторство

      • Сэмюэл Дж.

    Добавить комментарий

    Ваш адрес email не будет опубликован. Обязательные поля помечены *